Pharmacology Exam #3

Pataasin ang iyong marka sa homework at exams ngayon gamit ang Quizwiz!

Answer: d. Stevens-Johnson syndrome

A client is ordered to take trimethoprim-sulfamethoxazole (Bactrim). The nurse knows to expect which common adverse reaction? a. Bronchospasm b. Dysrhythmias c. Pseudomembranous colitis d. Stevens-Johnson syndrome

Answer: D

A client is prescribed dicloxacillin (Dynapen). The nurse plans to monitor the client for which side effect/adverse reaction? a.Seizures b.Renal failure c.Hypertension d.Hemolytic anemia

ANSWER: B → Saline laxatives are generally useful and safe for short-term treatment of constipation, cleansing the bowel prior to endoscopic examinations, and treating fecal impaction. However, they are not safe for frequent or prolonged usage.

You are interviewing the latest admission to your medical floor. The patient requests a magnesium citrate "cocktail" every night to "keep regular." Her requests is evidence of potential A. Constipation B. Laxative abuse C. Normal colon function D. Anorexia

Answer: B

Gentamycin is: A. Bacteriostatic B. Bactericidal C. Either

Answer: 3. Early morning Rationale: Corticosteroids (glucocorticoids) should be administered before 9:00 AM. Administration at this time helps minimize adrenal insufficiency and mimics the burst of glucocorticoids released naturally by the adrenal glands each morning. *Note the suffix "-sone," and recall that medication names that end with these letters are corticosteroids.*

138.) A daily dose of prednisone is prescribed for a client. A nurse reinforces instructions to the client regarding administration of the medication and instructs the client that the best time to take this medication is: 1. At noon 2. At bedtime 3. Early morning 4. Anytime, at the same time, each day

Answer: A

2.The nurse will monitor the client taking albuterol (Proventil) for which condition? a. Palpitations b. Hypoglycemia c. Bronchospasm d. Uterine contractions

Answer: D

A client receiving theophylline asks the nurse why a specific item was removed from the food tray. Which of the following is the best response? A. "Peas contain a chemical that can reduce the effects of the drug" B. "Brown beans are high in protein and can increase the effects of the drug" C. "Milk is high in calcium and increases the cardiac rate" D. "Coffee contains caffeine, which can increase the drug level more quickly"

Answer: 1. Insomnia

A client takes a dose of albuterol (Ventolin) prior to bedtime. Which effect would the nurse consider normal for this drug? 1. Insomnia 2. Sleepiness 3. Urticaria 4. Tinnitus

Answer: B

Achilles Tendon Rupture is listed as an adverse effect of ________. A. tetracyclines B. fluoroquinolones C. aminoglycosides D. penicillins

ANSWER: A → For patients in whom straining is potentially harmful or painful, stool softeners (e.g., docusate sodium) are the agents of choice.

An adult patient with hemerrhoids states that he experiences intense pain when straining to pass stools. What type of laxative would be the first choice? A. Stool softener B. Stimulant cathartic C. Saline cathartic D. Bulk-forming laxative

Answer: A

Azithromycin (Zithromax) is: A. Bacteriostatic B. Bactericidal C. Either

Answer: A Anticholinergic drugs are contraindicated in patients with glaucoma. These drugs can cause pupillary dilation, which can obstruct the flow of aqueous humor and increase intraocular pressure.

Before administering the anticholinergic drug scopolamine, the nurse would be careful to assess the patient for a history of which condition? A) Glaucoma B) Hyperthyroidism C) Rheumatoid arthritis D) Gastroenteritis

You can, but your patient will turn red.

Can you push Vancomycin quickly?

Answer: B

Cefazolin (Ancef) is: A. Bacteriostatic B. Bactericidal C. Either

Answer: B

Ciprofloxacin (Cipro) is: A. Bacteriostatic B. Bactericidal C. Either

Answer: C Rationale: Benylin is a drug included in most severe cold and flu preparations; therefore, the client must be instructed to seek medical attention should the drug become less effective with continued use.

Client teaching for clients on dextromethorphan (Benylin) should include: A. Reducing the dosage of the drug if insomnia occurs. B. Monitoring intake and output. C. Notifying the physician if the drug no longer seems effective. D. Continuing the drug at higher doses if the cough is not relieved after several days.

Answer: A

Discharge teaching to a patient receiving a beta-agonist bronchodilator should emphasize reporting which side effect? A. Tachycardia B. Nonproductive cough C. Hypoglycemia D. Sedation

C) Relief of nausea and vomiting. (Metoclopramide is classified as a prokinetic and antiemetic medication. If it is effective, the patient's nausea and vomiting should resolve).

Following administration of a dose of metoclopramide (Reglan) to the patient, the nurse determines that the medication has been effective when which of the following is noted? A) Decreased blood pressure B) Absence of muscle tremors C) Relief of nausea and vomiting D) No further episodes of diarrhea

Answer: A,B

How does penicillin (PCN) work to destroy bacteria? (Select all that apply.) A. PCN disrupts cell wall synthesis. B. PCN promotes cell wall destruction. C. PCN interferes with bacterial DNA processes. D. PCN interferes with bacterial RNA processes.

Answer: C

In general, penicillins are: A. Bactericidal B. Bacteriostatic C. Either

Answer: A

Isoniazid (INH), an antitubercular, is: A. Bacteriostatic B. Bactericidal C. Either

Answer: B

Levofloxacin (Levaquin) is: A. Bacteriostatic B. Bactericidal C. Either

Answer: C

Mast cell stabilizers are most effective in treating bronchoconstriction associated with which condition? A. Emphysema B. Exposure to cold C. Allergens D. Infection

Answer: A

Ondansetron (Zofran) has been ordered for the client undergoing cancer chemotherapy to control the severe nausea and vomiting. What side effects should the nurse observe for? A. Headache, dizziness, and fatigue B. Constipation, anxiety, and fever C. Abdominal cramping and irritability D. Psychosis and middle ear disturbances

Answer: D

Patient asks for an over-the-counter medication that will provide rapid relief of constipation. After ruling out possible contraindications, which drug would be most appropriate? A. psyllium (Metamucil) B. methylcellulose (Citrucel) C. docusate sodium (Colace) D. magnesium hydroxide (Milk of Magnesia)

Answer: b) acute adrenocortical insufficiency.

Prednisone (Deltasone) is prescribed to control inflammation in a client with interstitial lung disease. During client teaching, the nurse stresses the importance of taking prednisone exactly as prescribed and cautions against discontinuing the drug abruptly. A client who discontinues prednisone abruptly may experience: a) hyperglycemia and glycosuria. b) acute adrenocortical insufficiency. c) GI bleeding. d) restlessness and seizures.

Answer: A

Sulfasalazine (Azulfidine) has been ordered for a client. The nurse knows that this drug is most effective against which organisms? a.Escherichia coli and Clostridium b.Neisseria gonorrhoeae and H. Influenzae c.Pseudomonas aeruginosa and Helicobacter pylori d.Enterococcus faecium and Staphylococcus aureus

Answer: D Rationale: Amphotericin B (Fungizone) should be administered by slow intravenous infusion.

The health care provider has ordered amphotericin B (Fungizone) for the patient. The nurse recognizes that which is the most effective way to administer this medication to the patient? A Intravenously over 1 hour B Orally at regular intervals C By subcutaneous injection D Intravenously over 2 to 6 hours

Answer: D

The patient who is taking ciprofloxacin is complaining of lower leg pain or inflammation. What does the nurse suspect has happened? A. The patient has strained the ligaments in the leg. B. The patient has exercised at too high an intensity. C. The patient has overstretched the muscle in the calf. D. The patient has a tendon rupture.

Answer: D

Theophylline benefits COPD patient in many ways. Which of these statements is not true with regard to this medication? A. Reduces pulmonary artery pressure. B. Increases arterial oxygen tension C. Increases right ventricular function. D. Inhibits central respiratory activity.

Answer: B Rationale: Make sure to increase fluids, avoid antacids.

To maximize the absorption of tetracycline, the patient should avoid concurrent administration of: A. fluids B. antacids C. corticosteriods D. all of the above

Answer: A

Trimethorpim/Sulfamethoxazole or TMP/SMZ (Bactrim, Septra) is: A. Bacteriostatic B. Bactericidal C. Either

Answer: B

Vancomycin is: A. Bacteriostatic B. Bactericidal C. Either

Tooth discoloration

Why don't we give tetracyclines to kids?

Alcohol

You're gonna barf if you take metronidazole with ______.

ANSWER: B → Sufactant laxatives have little if any laxative effects. Their main value is to prevent straining while expelling stool. They usually act within 1 to 3 days and should be taken orally.

Your patient had a difficult delivery and is severely lacerated posterioirly. After surgical repair, she received postoperative instructions to increase fluids, medicate for pain as needed, and to medicate to ease defecation. Which laxative or cathartic did her obstretician likely recommend? A. Lubricant laxatives B. Surfactant laxatives C. Bulk-forming laxatives D. Saline laxatives

Answer: A

after a nebulizer treatment with the beta agonist abuterol, the patient complains of feeling a little "shaky" with slight tremors of the hands. The patients heart rate is 98 beats/min, increased from the pretreatment rate of 88 beats/min. The nurse knows that this reaction is an a. expected adverse effect of the medication b. allergic reaction to the medication c. indication that he has received an overdose of the medication d. idiosyncratic reaction to the medication

Answer: A,D,E Rationale: Tachycardia, nervousness, and headache may occur with the use of albuterol inhalers.

A client is receiving treatment for asthma with albuterol (Proventil, VoSpire). The nurse teaches the client that while serious adverse effects are uncommon, the following may occur. (Select all that apply). a. Tachycardia b. Sedation c. Temporary dyspnea d. Nervousness e. Headache

Answer: B

A client is taking a sulfonamide for an acute urinary tract infection. Which medication does the nurse realize is a short-acting sulfonamide? a.sulfasalazine (Azulfidine) b.sulfadiazine (Microsulfon) c.sulfamethoxazole (Gantanol) d.co-trimoxazole/TMP-SMZ (Bactrim)

Answer: A Rationale: Tachycardia is a response to activation of beta1 receptors. It can occur when large doses of albuterol are used and selectivity decreases. The nurse should question the patient about the number of inhalations used. Isoproterenol can cause hyperglycemia in diabetic patients. Isoproterenol has more side effects than albuterol. Tremors are an expected side effect and are not an indication for stopping the drug.

136. A patient with asthma uses albuterol (Ventolin) for wheezing. The nurse assesses the patient and notes vital signs of HR, 96 beats per minute; RR, 18 breaths per minute; and BP, 116/78 mm Hg. The patient has clear breath sounds and hand tremors. What will the nurse do? a. Ask the patient how often the inhaler is used. b. Check the patient's blood glucose level. c. Request an order for isoproterenol (Isuprel) to reduce side effects. d. Stop the medication and report the tremors to the provider.

Answer: 2. Coffee, cola, and chocolate Rationale: Theophylline is a xanthine bronchodilator. The nurse teaches the client to limit the intake of xanthine-containing foods while taking this medication. These include coffee, cola, and chocolate.

146.) A client has begun therapy with theophylline (Theo-24). The nurse tells the client to limit the intake of which of the following while taking this medication? 1. Oranges and pineapple 2. Coffee, cola, and chocolate 3. Oysters, lobster, and shrimp 4. Cottage cheese, cream cheese, and dairy creamers

2. Take the daily dose at bedtime. Rationale: The client taking a single daily dose of theophylline, a xanthine bronchodilator, should take the medication early in the morning. This enables the client to have maximal benefit from the medication during daytime activities. In addition, this medication causes insomnia. The client should take in at least 2 L of fluid per day to decrease viscosity of secretions. The client should check with the physician before changing brands of the medication. The client also checks with the HCP before taking OTC cough, cold, or other respiratory preparations because they could cause interactive effects, increasing the side effects of theophylline and causing dysrhythmias.

147.) A client with a prescription to take theophylline (Theo-24) daily has been given medication instructions by the nurse. The nurse determines that the client needs further information about the medication if the client states that he or she will: 1. Drink at least 2 L of fluid per day. 2. Take the daily dose at bedtime. 3. Avoid changing brands of the medication without health care provider (HCP) approval. 4. Avoid over-the-counter (OTC) cough and cold medications unless approved by the HCP.

Answer: A Rationale: Peripheral neurotoxicity associated can be prevented by taking vitamin B6 when being treated with INH. Visual changes, hearing problems, and orange staining are adverse effects of other TB medications.

15. A patient is receiving isoniazid (INH) after having a positive tuberculin skin test. Which information will the nurse include in the patient teaching plan? a. "Take vitamin B6 daily to prevent peripheral nerve damage." b. "Read a newspaper daily to check for changes in vision." c. "Schedule an audiometric examination to monitor for hearing loss." d. "Avoid wearing soft contact lenses to avoid orange staining."

Answer: 3. Liver enzyme levels Rationale: INH therapy can cause an elevation of hepatic enzyme levels and hepatitis. Therefore, liver enzyme levels are monitored when therapy is initiated and during the first 3 months of therapy. They may be monitored longer in the client who is greater than age 50 or abuses alcohol.

2.) A client with tuberculosis is being started on antituberculosis therapy with isoniazid (INH). Before giving the client the first dose, a nurse ensures that which of the following baseline studies has been completed? 1. Electrolyte levels 2. Coagulation times 3. Liver enzyme levels 4. Serum creatinine level

Answer: A,C,D,E,F

A client is taking azithromycin (Zithromax). The nurse should apply which interventions? (Select all that apply.) a.Monitor periodic liver function tests. b.Dilute with 50 mL for IV administration. c.Tell the client to report any hearing loss. d.Instruct the client to report evidence of superinfection. e.Teach the client to take oral drug 1 hour a.c. or 2 hours p.c. f.Avoid antacids from 2 hours prior to 2 hours after azithromycin administration.

Answer: A Rationale: A large initial dose is often used as a loading dose to help get serum drug levels to plateau levels more quickly. Larger doses do not prevent first pass effects in drugs susceptible to this type of metabolism. Tapering of doses sometimes is used to prevent rebound or withdrawal effects and is done by stepping down the amount of drug with each dose. Tubular reabsorption is a process that allows drugs to be reabsorbed from the urine into the blood.

241. A patient is given a prescription for azithromycin (Zithromax) and asks the nurse why the dose on the first day is twice the amount of the dose on the next 4 days. Which reply by the nurse is correct? a. "A large initial dose helps to get the drug to optimal levels in the body faster." b. "The first dose is larger to minimize the first pass effect of the liver." c. "The four smaller doses help the body taper the amount of drug more gradually." d. "Tubular reabsorption is faster with initial doses, so more is needed at first."

Answer: D Rationale: Penicillin G is inactivated by digestive enzymes in the stomach and cannot be given orally. It does not have a narrow therapeutic range.

243. The nurse is preparing to administer penicillin G intramuscularly to a child. The child's parents ask why the drug cannot be given in an oral liquid form. What is the nurse's reply? a. "This drug causes severe gastric upset if given orally." b. "This drug has a narrow therapeutic range, and the dose must be tightly controlled." c. "This drug is absorbed much too quickly in an oral form." d. "This drug would be inactivated by enzymes in the stomach."

Answer: D Rationale: It is not within the nurse's scope of practice to change the dose of a medication without an order from a prescriber. Asking the patient to demonstrate inhaler use helps the nurse to evaluate the patient's ability to administer the medication properly and is part of the nurse's evaluation. Assessing tobacco smoke exposure helps the nurse determine whether nondrug therapies, such a smoke avoidance, can be used as an adjunct to drug therapy. Performing a physical assessment helps the nurse evaluate the patient's response to the medication.

255. A patient is using a metered-dose inhaler containing albuterol for asthma. The medication label instructs the patient to administer "2 puffs every 4 hours as needed for coughing or wheezing." The patient reports feeling jittery sometimes when taking the medication, and she doesn't feel that the medication is always effective. Which is not an appropriate nursing intervention for this patient? a. Asking the patient to demonstrate use of the inhaler b. Assessing the patient's exposure to tobacco smoke c. Auscultating lung sounds and obtaining vital signs d. Suggesting that the patient use one puff to reduce side effects

Answer: D Rationale: Pruritus and wheezing are signs of a possible allergic reaction, which can be fatal; therefore, the medication should not be given and the prescriber should be notified. When patients are having a potentially serious reaction to a medication, the nurse should not continue giving the medication. Antihistamines may help the symptoms of an allergic reaction, but the first priority is to stop the medication. Obtaining a chest radiograph is not helpful.

264. A patient has been receiving intravenous penicillin for pneumonia for several days and begins to complain of generalized itching. The nurse auscultates bilateral wheezing and notes a temperature of 38.5° C (101° F). Which is the correct action by the nurse? a. Administer the next dose and continue to evaluate the patient's symptoms. b. Ask the prescriber if an antihistamine can be given to relieve the itching. c. Contact the prescriber to request an order for a chest radiograph. d. Hold the next dose and notify the prescriber of the symptoms.

Answer: C Rationale: Combining bronchodilators improves effectiveness. Atrovent does not have to be used before Proventil, it does not decrease airway inflammation, and it does not require hospitalization.

28. A patient with COPD tells the nurse, "At home, I only have to use an albuterol (Proventil) inhaler. Why did the doctor add an ipratropium (Atrovent) inhaler while I'm in the hospital? The appropriate response by the nurse is a. "Atrovent will dilate the airways and allow the Proventil to penetrate more deeply." b. "Atrovent is being used to decrease airway inflammation and sputum production." c. "Atrovent works differently to dilate the bronchi, and the two drugs together are more effective." d. "Atrovent is a potent bronchodilator and patients need to be hospitalized when receiving it."

Answer: 1. "I can take aspirin or my antihistamine if I need it." Rationale: Aspirin and other over-the-counter medications should not be taken unless the client consults with the health care provider (HCP). The client needs to take the medication at the same time every day and should be instructed not to stop the medication. A slight weight gain as a result of an improved appetite is expected, but after the dosage is stabilized, a weight gain of 5 lb or more weekly should be reported to the HCP. Caffeine-containing foods and fluids need to be avoided because they may contribute to steroid-ulcer development.

31.) A community health nurse visits a client at home. Prednisone 10 mg orally daily has been prescribed for the client and the nurse reinforces teaching for the client about the medication. Which statement, if made by the client, indicates that further teaching is necessary? 1. "I can take aspirin or my antihistamine if I need it." 2. "I need to take the medication every day at the same time." 3. "I need to avoid coffee, tea, cola, and chocolate in my diet." 4. "If I gain more than 5 pounds a week, I will call my doctor."

Answer: D Rationale: Albuterol is a rapidly acting bronchodilator and is the first-line medication to reverse airway narrowing in acute asthma attacks. The other medications work more slowly.

34. All of these orders are received for a patient having an acute asthma attack. Which one will the nurse administer first? a. IV methylprednisolone (Solu-Medrol) 60 mg b. triamcinolone (Azmacort) 2 puffs per MDI c. salmeterol (Serevent) 50 mcg per DPI d. albuterol (Ventolin) 2.5 mg per nebulizer

Answer: 2 Rationale: A common side effect of the TB drug INH is peripheral neuritis. This is manifested by numbness, tingling, and paresthesias in the extremities. This side effect can be minimized by pyridoxine (vitamin B6) intake. Options 1, 3, and 4 are incorrect.

47.) A client has been taking isoniazid (INH) for 2 months. The client complains to a nurse about numbness, paresthesias, and tingling in the extremities. The nurse interprets that the client is experiencing: 1. Hypercalcemia 2. Peripheral neuritis 3. Small blood vessel spasm 4. Impaired peripheral circulation

Answer: 2. Report yellow eyes or skin immediately. Rationale: INH is hepatotoxic, and therefore the client is taught to report signs and symptoms of hepatitis immediately (which include yellow skin and sclera). For the same reason, alcohol should be avoided during therapy. The client should avoid intake of Swiss cheese, fish such as tuna, and foods containing tyramine because they may cause a reaction characterized by redness and itching of the skin, flushing, sweating, tachycardia, headache, or lightheadedness. The client can avoid developing peripheral neuritis by increasing the intake of pyridoxine (vitamin B6) during the course of INH therapy for TB.

48.) A client is to begin a 6-month course of therapy with isoniazid (INH). A nurse plans to teach the client to: 1. Drink alcohol in small amounts only. 2. Report yellow eyes or skin immediately. 3. Increase intake of Swiss or aged cheeses. 4. Avoid vitamin supplements during therapy.

4. Sore throat Rationale: Clients taking trimethoprim-sulfamethoxazole (TMP-SMZ) should be informed about early signs of blood disorders that can occur from this medication. These include sore throat, fever, and pallor, and the client should be instructed to notify the health care provider if these symptoms occur. The other options do not require health care provider notification.

66.) Trimethoprim-sulfamethoxazole (TMP-SMZ) is prescribed for a client. A nurse should instruct the client to report which symptom if it developed during the course of this medication therapy? 1. Nausea 2. Diarrhea 3. Headache 4. Sore throat

Answer: B Rationale: Cromolyn is prescribed to reduce airway inflammation. It takes several weeks for maximal effect and is not used to treat acute asthma symptoms Albuterol is used as a rescue medication in mild persistent asthma and will not decrease inflammation.

8. A patient who has mild persistent asthma uses an albuterol (Proventil) inhaler for chest tightness and wheezing has a new prescription for cromolyn (Intal). To increase the patient's management and control of the asthma, the nurse should teach the patient to a. use the cromolyn when the albuterol does not relieve symptoms. b. use the cromolyn to prevent inflammatory airway changes. c. administer the cromolyn first for chest tightness or wheezing. d. administer the albuterol regularly to prevent airway inflammation.

Answer: D Rationale: Drinking alcohol can induce isoniazid-related hepatitis. If hepatic damage occurs, the client's urine may become dark and appear concentrated. GI upset frequently occurs when isoniazid is taken on an empty stomach, so taking this drug with meals decreases GI upset. The client should avoid taking aluminum-containing antacids, such as aluminum hydroxide, with isoniazid as it may decrease the drug's effects.

9. The nurse is preparing to teach a client about the effects of isoniazid (INH). Which information is important for the client to understand? A. Isoniazid should be taken on an empty stomach. B. Prolonged use of isoniazid produces poorly concentrated urine. C. Taking aluminum hydroxide (Maalox)® with isoniazid minimizes gastrointestinal upset. D. Drinking alcohol daily can increase the incidence of drug-induced hepatitis.

Answer: B Rationale: This medication may decrease the effectiveness of oral contraceptives. The nurse needs to assess whether or not the patient is on oral contraceptives and whether or not the patient is sexually active. Awarded 1.0 points out of 1.0 possible points.

A 22-year-old female patient is put on amoxicillin. Which is the most important intervention for this patient? A Instruct the patient to not take the medication before meals. B Assess if the patient is on oral contraceptives. C Inform the patient about possible superinfections. D Assess the patient for cross sensitivity.

Answer: B

A 33 year old pt is in the outpatient cancer center for his first round of chemotherapy. The nurse knows that which schedule is the most appropriate timing for the intravenous antiemetic drug? A. Four hours before the chemotherapy begins B. Thirty minutes before the chemotherapy begins C. At the same time as the chemotherapy begins D. At the first signs of nausea

Answer: A Rationale: Atrovent is contraindicated in patients with hypersensitivity to soys lechtin or related food products such as soybean and peanut.

A 65 year old client is prescribed ipratropium (Atrovent) for the treatment of asthma. Which of the following conditions should be reported to the HCP before giving the client the drug? a. A reported allergy to peanuts b. A history of intolerance to albuterol c. A history of bronchospasms d. A reported allergy to chocolate

ANSWER: C → Long-term use of PPIs is associated with increased risk of hip fractures in people older than 50 years of age; the risk of fractures increases the longer the medications are taken and is greater in people who take higher dosages of PPIs. Bone density assessment may this be warranted.

A 72-year-old patient with a history of peptic ulcer disease has been taking a PPI for the past 3 years. In light of the possible adverse effects of long-term PPI use, the nurse practitioner should focus on what assessment? A. Joint mobility B. Integumentary assessment C. Bone density D. Dental health

c. Omeprazole (Prilosec) There is a potential link between proton pump inhibitors (PPIs) (e.g., omeprazole) use and bone metabolism. Long-term use or high doses of PPIs may increase the risk of fractures of the hip, wrist, and spine. Lower doses or shorter duration of therapy should be considered.

A 74-year-old female patient with gastroesophageal reflux disease (GERD) takes over-the-counter medications. For which medication, if taken long-term, should the nurse teach about an increased risk of fractures? a. Sucralfate (Carafate) b. Cimetidine (Tagamet) c. Omeprazole (Prilosec) d. Metoclopramide (Reglan)

Answer: 3. A β2 agonist Rationale: In treating an acute asthma attack, a short acting β2 agonist such as albuterol (Proventil HFA) will be given to produce bronchodilation. Options 1, 2, and 4 are long-term control (preventive) medications.

A child is brought to the emergency department for treatment of an acute asthma attack. The nurse prepares to administer which of the following medications first? 1. Oral corticosteroids 2. A leukotriene modifier 3. A β2 agonist 4. A nonsteroidal anti-inflammatory

Answer: D

A client is taking docusate sodium (Colace). The nurse monitors which of the following to determine whether the client is having a therapeutic effect from this medication? A. Abdominal pain B. Reduction in steatorrhea C. Hematest-negative stools D. Regular bowel movements

Answer: D

A client is taking levofloxacin (Levaquin). The nurse knows that which is true regarding this drug? a. Administered IV only b. May cause hypertension c. Classified as an aminoglycoside d. Adverse reaction includes dysrhythmias

a. Singulair decreases inflammation and mucus secretion Rationale: Leukotriene modifiers such as montelukast block the action of leukotrienes, and therefore decrease mucous secretion and reduce inflammation, preventing bronchoconstriction. Montelukast blocks the action of leukotrienes and decreases mucous secretion. Leukotriene modifiers such as montelukast do not prevent smooth muscle contraction; they decrease mucous secretion and reduce inflammation. Leukotrienes are released when a client is exposed to an allergen. Leukotriene modifiers such as montelukast do not protect the airway from effects of allergen exposure.

A client beginning medication therapy with montelukast (Singulair) asks the nurse how the medication is helping the symptoms. Which is the nurse's best response? a. "Singulair decreases inflammation and mucus secretion." b. "Singulair increases mucus secretion and bronchodilation" c. "Singulair prevents smooth muscle contraction by nervous system stimulation." d. "Singulair protects the airway from the effects of allergen exposure."

Rinse his mouth with water after each use. Rationale: Flunisolide (AeroBid) is an inhaled corticosteroid. Rinsing the mouth will help prevent oral candidal infections. It is not used to treat an acute asthma attack and should be taken with the client's bronchodilator medications. If the client is taking oral prednisone, it needs to be tapered off to prevent acute adrenal crisis because flunisolide is minimally absorbed systemically.

A client demonstrates understanding of flunisolide (AeroBid) by saying that he will do what?

Answer: D

A client entering the medicine clinic has been diagnosed with an overactive bladder. Which medication would the nurse expect to be ordered? A. Ciprofloxacin (Cipro) B. Oxybutynin (Ditropan) C. Bethanechol (Urecholine) D. Tolterodine tartrate (Detrol)

Answer: C Loperamide (Imodium) is an antidiarrheal

A client has a PRN order for loperamide (Imodium). The nurse should plan to administer this medication if the client has: A. constipation B. abdominal pain C. episode of diarrhea D. Hematest-positive nasogastric tube drainage

Answer: 4. Administering the albuterol before the beclomethasone dipropionate

A client has a prescription to receive albuterol (Proventil HFA) two puffs and beclomethasone dipropionate (Qvar) two puffs by metered-dose inhaler. The nurse plans to give these medications most effectively by: 1. Alternating a single puff of each hourly, beginning with the beclomethasone dipropionate 2. Alternating a single puff of each hourly, beginning with the albuterol 3. Administering the beclomethasone dipropionate before the albuterol 4. Administering the albuterol before the beclomethasone dipropionate

Answer: A,D,E,F

A client has been diagnosed with tuberculosis and is to begin the antitubercular medications isoniazid, rifampin, and ethambutol. What should the nurse do? (Select all that apply.) a.Encourage periodic eye examinations. b.Instruct client to take medications with meals. c.Suggest that client take antacids with medications to prevent GI distress. d.Advise client to report numbness and tingling of hands or feet. e.Alert client that body fluids may develop a red-orange color. f.Teach client to avoid direct sunlight and to use sunblock.

C For exercise related issues, Singulair should be taken at least 2 hours before exercise. It is a leukotrine blocker for asthma that, recent data indicates, can have psychiatric issues.

A client has been prescribed Montelukast (Singulair). What information provided by the nurse would NOT be correct? A. It is a leukotrine blocker B. An uncommon side effect is psychiatric issues C. If used, it should be taken at least 1 hour before exercise D. It can be used in a child as young as 2

Answer: 2,3 Rationale: Clients who are taking nitrofurantoin (Macrodantin) should be instructed to take the medication with meals and to increase their fluid intake to minimize gastrointestinal distress. The urine may become brown in color. Although this change is harmless, clients need to be prepared for this color change. The client should be instructed to take the full prescription and not to stop taking the drug because symptoms have subsided. The medication should not be taken with antacids as this may interfere with the drug's absorption.

A client has been prescribed nitrofurantoin (Macrodantin) for treatment of a lower urinary tract infection. Which of the following instructions should the nurse include when teaching the client how to take this medication? Select all that apply. 1. "Take the medication on an empty stomach." 2. "Your urine may become brown in color." 3. "Increase your fluid intake." 4. "Take the medication until your symptoms subside." 5. "Take the medication with an antacid to decrease gastrointestinal distress."

Absence of dysuria

A client has been receiving nitrofurantoin sodium (Macrodantin). The nurse determines that the therapy is effective if which of the following is noted?

Answer: B Rationale: Heartburn ,omeprazole (Prilosec) is used for GERD

A client has been taking omeprazole (Prilosec) for 4 weeks. The nurse determines that the client is receiving the optimal intended effect of the medication if the client reports absence of which of the following symptoms? A. Diarrhea B. Heartburn C. Flatulence D. Constipation

Answer: A

A client is beginning isoniazid and rifampin treatment for tuberculosis. The nurse gives the client which instruction? a.Do not skip doses. b.Take the drugs t.i.d. with food. c.Take an antacid with the drugs to decrease GI distress. d.Take rifampin initially, and then begin isoniazid after 2 months.

Answer: D

A client is ordered to take trimethoprim-sulfamethoxazole (Bactrim). The nurse knows to expect which common adverse reaction? a. Bronchospasm b. Dysrhythmias c. Pseudomembranous colitis d. Stevens-Johnson syndrome

"Take the ipratropium at least 5 minutes before the cromolyn." Rationale:When using an anticholinergic in conjunction with an inhaled glucocorticoid or cromolyn, the ipratropium should be used 5 minutes before the steroid. This causes the bronchioles to dilate so the steroid or cromolyn can get deeper into the lungs.

A client is prescribed ipratropium and cromolyn sodium. What will the nurse teach the client?

Answer; B

A client is prescribed theophylline to relax the smooth muscles of the bronchi. The nurse monitors the client's theophylline serum levels to maintain which therapeutic range? a. 1 to 10 mcg/mL b. 10 to 20 mcg/mL c. 20 to 30 mcg/mL d. 30 to 40 mcg/mL

Answer: C

A client is receiving amoxicillin (Amoxil). The nurse knows that the action of this drug is by which process? a.Inhibition of protein synthesis b.Alteration of membrane permeability c.Inhibition of bacterial cell-wall synthesis d.Alteration of synthesis of bacterial ribonucleic acid

Answer: A

A client is taking sulfasalazine (Azulfidine). What should the nurse teach the client to do? a.Drink at least 10 glasses of fluid per day. b.Monitor blood glucose carefully to avoid hyperglycemia. c.Avoid operating a motor vehicle as this drug may cause drowsiness. d.Take this drug with an antacid to decrease the risk of gastrointestinal distress.

Answer: C RATIONALE: Administering oral doses with food (C) helps minimize GI discomfort. (A) would be appropriate only if changing the time of the dose corresponds to meal times while at the same time maintaining an appropriate time interval between doses. (B) would disrupt the dosing schedule, and could result in a nontherapeutic serum level of the medication. (D) should not be attempted before other interventions, such as (C), have been proven ineffective in relieving the nausea.

A client receiving albuterol (Proventil) tablets complains of nausea every evening with her 9 p.m. dose. What action should the nurse take to alleviate this side effect? A) Change the time of the dose. B) Hold the 9 p.m. dose. C) Administer the dose with a snack. D) Administer an antiemetic with the dose.

Answer: 4

A client receiving nitrofurantoin (Macrodantin) calls the health care provider's office complaining of side effects related to the medication. Which side effect indicates the need to stop treatment with this medication? 1. Nausea 2. Diarrhea 3. Anorexia 4. Cough and chest pain

Rationale: (Drink increased amounts of fluids every day.) A client should drink increased fluids (2000 to 3000 mL/day) to decrease viscosity and increase expectoration of secretions. This is standard advice for clients receiving any of the adrenergic bronchodilators, unless the client has another health problem that contraindicates an increased fluid intake. Additional exercise will not effectively clear bronchial secretions. A dehumidifier will dry secretions, making the situation worse. The client should not take additional medication.

A client taking albuterol (ProAir HFA) by inhalation cannot cough up secretions. What should the nurse suggest that the client do to assist in expectoration of secretions?

Answer: C

A client taking an oral theophylline preparation is due for her next dose and has a blood pressure of 100/50 mm Hg and a heart rate of 110. The client is irritable. What is the best action for the nurse to take? a. Continue to monitor the client. b. Call the health care provider. c. Hold the next dose of theophylline. d. Administer oxygen 2 lpm via nasal cannula.

Answer: B

A client taking isoniazid is worried about the side effects/adverse reactions. The nurse realizes that which is a common adverse reaction of isoniazid? a.Ototoxicity b.Hepatotoxicity c.Nephrotoxicity d.Optic nerve toxicity

Answer: 3 Use of oral inhalant corticosteroids such as flunisolide (AeroBid) can lead to the development of oral thrush, a fungal infection. Once developed, thrush must be treated by antifungal therapy; it will not resolve on its own. Fungal infections can develop even without overuse of the corticosteroid inhaler. Although good oral hygiene can help prevent development of a fungal infection, it cannot be used alone to treat the problem.

A client who has been taking flunisolide (AeroBid), two inhalations a day, for treatment of asthma.has painful, white patches in his mouth. Which response by the nurse would be most appropriate? 1. "This is an anticipated adverse effect of your medication. It should go away in a couple of weeks." 2. "You are using your inhaler too much and it has irritated your mouth." 3. "You have developed a fungal infection from your medication. It will need to be treated with an antifungal agent." 4. "Be sure to brush your teeth and floss daily. Good oral hygiene will treat this problem."

Answer: 3 Rationale: Use of oral inhalant corticosteroids such as flunisolide (AeroBid) can lead to the development of oral thrush, a fungal infection. Once developed, thrush must be treated by antifungal therapy; it will not resolve on its own. Fungal infections can develop even without overuse of the corticosteroid inhaler. Although good oral hygiene can help prevent development of a fungal infection, it cannot be used alone to treat the problem.

A client who has been taking flunisolide (AeroBid), two inhalations a day, for treatment of asthma.has painful, white patches in his mouth. Which response by the nurse would be most appropriate? 1. "This is an anticipated adverse effect of your medication. It should go away in a couple of weeks." 2. "You are using your inhaler too much and it has irritated your mouth." 3. "You have developed a fungal infection from your medication. It will need to be treated with an antifungal agent." 4. "Be sure to brush your teeth and floss daily. Good oral hygiene will treat this problem."

At the low end of the therapeutic range

A client who is taking theophylline (Theo-24) reports to the health care clinic as scheduled to have a serum theophylline level drawn. The blood is drawn and the laboratory calls the nurse and reports the results as 10 mcg/mL. The client asks the nurse what this figure means. The nurse accurately responds by stating this level is:

Answer: a) question the order because the dosage is too low.

A client who weighs 175 lb (79.4 kg) is receiving aminophylline (Aminophyllin) (400 mg in 500 ml) at 50 ml/hour. The theophylline level is reported as 6 mcg/ml. The nurse calls the physician who instructs the nurse to change the dosage to 0.45 mg/kg/hour. The nurse should: a) question the order because the dosage is too low. b) question the order because the dosage is too high. c) set the pump at 45 ml/hour. d) stop the infusion and have the laboratory repeat the theophylline measurement.

Answer: A

A client with COPD is taking a leukotriene antagonist, montelukast (Singulair). The nurse is aware that this medication is given for which purpose? a. Maintenance treatment of asthma b. Treatment of an acute asthma attack c. Reversing bronchospasm associated with COPD d. Treatment of inflammation in chronic bronchitis

a. Maintenance treatment of asthma

A client with COPD is taking a leukotriene antagonist, montelukast (Singulair). The nurse is aware that this medication is given for which purpose? a. Maintenance treatment of asthma b. Treatment of an acute asthma attack c. Reversing bronchospasm associated with COPD d. Treatment of inflammation in chronic bronchitis

Answer: C

A client with a diagnosis of intestinal amebiasis develops severe nausea, vomiting, fever, facial flushing, slurred speech, tachycardia, hypotension, and palpitations. A beginning assessment reveals that the client has just had several alcoholic beverages. The nurse should obtain a drug history for which drug? a. bacitracin (Baci-IM) b. fluconazole (Diflucan) c. metronidazole (Flagyl) d. ethambutol (Myambutol)

Answer: D

A client with a gastric ulcer has an order for sucralfate (Carafate), 1 g orally four times a day. The nurse schedules the medications for which of the following times? A. With meals and at bedtime B. Every 6 hours around the clock C. One hour after meals and at bedtime D. One hour before meals and at bedtime

Answer: D

A client with a lower urinary tract infection has been prescribed nitrofurantoin (Macrodantin). What side effects/adverse reactions would the nurse teach the client to expect? A. Irritability B. Anxiety C. Crystalluria D. Brown, discolored urine

Answer: 3 Rationale: Antibiotics have the maximum effect when a blood level of the medication is maintained. However, because nitrofurantoin (Macrodantin) is readily absorbed from the gastrointestinal tract and is primarily excreted in urine, toxicity may develop by doubling the dose. The client should not skip a dose if she realizes that she has missed one. Additional fluids, especially water, should be encouraged, but not forced to promote elimination of the antibiotic from the body. Adequate fluid intake aids in the prevention of urinary tract infections, in addition to an acidic urine.

A client with a urinary tract infection is to take nitrofurantoin (Macrodantin) four times each day. The client asks the nurse, "What should I do if I forget a dose?" What should the nurse tell the client? 1. "You can wait and take the next dose when it is due." 2. "Double the amount prescribed with your next dose." 3. "Take the prescribed dose as soon as you remember it, and if it is very close to the time for the next dose, delay that next dose." 4. "Take a lot of water with a double amount of your prescribed dose."

loratadine should be taken once daily for allergic rhinitis.

A client with allergic rhinitis is prescribed loratadine (Claritin). On a follow-up visit, the client tells the nurse, "I take one 10-mg tablet of Claritin with a glass of water two times daily." The nurse concludes that the client requires additional teaching about this medication because:

A) Albuterol is a beta2 agonist that acts rapidly as a bronchodilator.

A client with asthma has pneumonia, is reporting increased shortness of breath, and has inspiratory and expiratory wheezes. All of these medications are prescribed. Which medication should the nurse administer first? A. Albuterol (Proventil) 2 inhalations B. Fluticasone (Flovent) 2 inhalations C. Ipratropium (Atrovent) 2 inhalations D. Salmeterol (Serevent) 2 inhalations

Answer: C Rationale: Clients who are allergic to penicillin are given a 14-day regimen of tetracycline or doxycycline. Acyclovir is used to treat genital herpes. Ceftriaxone may be used for gonorrhea. Podophyllum resin is used to treat genital warts.

A client with primary syphilis is allergic to penicillin. The nurse would expect the physician to order which agent? a) Podophyllum resin b) Tetracycline c) Ceftriaxone d) Acyclovir

Answer: 4

A cromolyn sodium (Intal) inhaler is prescribed for a client with allergic asthma. A nurse provides instructions regarding the side effects of this medication. The nurse tells that client that which undesirable effect is associated with this medication? 1. Insomnia 2. Constipation 3. Hypotension 4. Bronchospasm

Answer: D

A female patient will receive doxycycline to treat a sexually transmitted illness (STI). What information will the nurse include when teaching this patient about this medication? a. Nausea and vomiting are uncommon adverse effects. b. The drug may cause possible teratogenic effects. c. Increase intake of dairy products with each dose of this medication. d. Use a backup method of contraception if taking oral contraceptives.

Answer: C

A gardener needs a decongestant because of seasonal allergy problems and asks the nurse whether he should take an oral form or a nasal spray. The nurse's answer considers that one benefit of orally administered decongestants is A. immediate onset. B. a more potent effect. C. lack of rebound congestion. D. shorter duration.

Answer: 1,2,3,4 Rationale: 5 and 6 are PPI's (proton pump inhibitors)

A histamine (H2) - receptor antagonist will be prescribed for a client. The nurse understands that which medications are H2- receptor antagonists, one of which could be prescribed. Select all that apply. 1. Nizatidine (Axid) 2. Ranitidine (Zantac) 3. Famotidine (Pepcid) 4. Cimetidine (Tagamet) 5. Esomeprazole (Nexium) 6. Lansoprazole (Prevacid)

Answer: A Rationale: To avoid a hypotensive reaction from rapid I.V. administration, the nurse should infuse vancomycin slowly, over 60 to 90 minutes, in a large volume of fluid. Although neutropenia may occur in approximately 5% to 10% of clients receiving vancomycin, this adverse effect reverses rapidly when the drug is discontinued.

A male client is to receive I.V. vancomycin (Vancocin). When preparing to administer this drug, the nurse should keep in mind that: a. vancomycin should be infused over 60 to 90 minutes in a large volume of fluid. b. vancomycin may cause irreversible neutropenia. c. vancomycin should be administered rapidly in a large volume of fluid. d. vancomycin should be administered over 1 to 2 minutes as an I.V. bolus.

Answer: D) Promethazine (Phenergan) A common adverse effect of promethazine, an antihistamine antiemetic agent, is dry mouth; another is blurred vision.

A patient reports having dry mouth and asks for some liquid to drink. The nurse reasons that this symptom can most likely be attributed to a common adverse effect of which of the following medications? A) Digoxin (Lanoxin) B) Cefotetan (Cefotan) C) Famotidine (Pepcid) D) Promethazine (Phenergan)

Answer A. Rationale: The therapeutic range for the serum theophylline level is 10 to 20 mcg/mL. If the level is below the therapeutic range, the client may experience frequent exacerbations of the disorder. Although all the options identify values within the therapeutic range, option A is the option that reflects a need for compliance with medication.

A male client who takes theophylline for chronic obstructive pulmonary disease is seen in the urgent care center for respiratory distress. Once the client is stabilized, the nurse begins discharge teaching. The nurse would be especially vigilant to include information about complying with medication therapy if the client's baseline theophylline level was: a. 10 mcg/mL b. 12 mcg/mL c. 15 mcg/mL d. 18mcg/mL

Answer: C RATIONALE: Beta-1 agonists improve cardiac output by increasing the heart rate and blood pressure and are indicated in heart failure (C), shock, atrioventricular block dysrhythmias, and cardiac arrest. Glaucoma (A) is managed using adrenergic agents and beta-adrenergic blocking agents. Beta-1 blocking agents are used in the management of hypertension (B). Medications that stimulate beta-2 receptors in the bronchi are effective for bronchoconstriction in respiratory disorders, such as asthma (D).

A medication that is classified as a beta-1 agonist is most commonly prescribed for a client with which condition? A) Glaucoma. B) Hypertension. C) Heart failure. D) Asthma.

ANSWER: C → Omeprazole increases blood levels of warfarin; this creates a risk for bleeding.

A nurse at a long-term care facility is reviewing a new resident's medication regimen and notes that he takes omeprazole and warfarin. The nurse should be aware that this combination of medication creates a risk of A. Rebound GERD B. Venous thromboembolism C. Hemorrhage D. Fluid volume excess

Answer: A, B Rationale: Health care providers commonly give polyethylene glycol-electrolyte solution and bisacodyl to evacuate the colon in preparation for a colonoscopy.

A nurse is caring for a patient preparing to undergo a colonoscopy. Which of the following drugs do health care providers commonly use to enhance bowel cleansing? (Choose all that apply.) A. Polyethylene glycol-electrolyte solution (GoLYTELY) B. Bisacodyl (Dulcolax) C. Psyllium (Metamucil) D. Docusate sodium (Colace)

Answer: B. 1 hr before meals Rationale: Sucralfate is a mucosal protectant, the client should take it on an empty stomach, 1 hr before meals, for maximum effectiveness

A nurse is talking with a client who has peptic ulcer disease and is starting therapy with sucralfate (Carafate). The nurse should instruct the client to take this medication: A. with antacid B. 1 hr before meals C. with food or milk D. immediatly after meals

In the middle of the therapeutic range

A nurse notes that the result of the serum theophylline level of a client taking theophylline is 15 mcg/mL. The nurse interprets this result as:

ANSWER: A → Laxatives and cathartics should not be used in the presence of undiagnosed abdominal pain or other signs of intestinal obstruction due to the risk of perforation and peritonitis. Vitamin supplements, hemorrhoids, and decreased mobility do not necessarily contraindicate the use of laxatives.

A nurse who works with elderly adults in the community has often recommended the use of bulk-forming laxatives to patients. In which of the following circumstances would the use of a laxative be contraindicated? A. A patient is experiencing acute abdominal pain. B. A patient has a history of hemorhhoids. C. A patient is taking vitamin B supplements. D. A patient is unable to ambulate to the bathroom without assistance.

Administer with a full glass of water and encourage fluid intake of 1500-2000 mL/day to prevent crystalluria

A nursing intervention for administering Ciprofloxacin to a pt is :

Answer: A Rationale: Although albuterol affects the beta2 receptors, at higher doses it may also activate beta1 receptors, causing an increase in the heart rate. Bronchodilation does not lower the blood pressure, and activation of beta1 receptors actually may increase it. Tachycardia is a common side effect. The inhaled route reduces the likelihood of systemic side effects.

A nursing student asks why albuterol, which is selective for beta2 receptors, causes an increased heart rate in some patients. How does the nurse respond? a. "Adrenergic agonists can lose their selectivity when given at higher doses." b. "Bronchodilation lowers blood pressure, which causes a reflex tachycardia." c. "Some patients metabolize the drug differently and have unusual side effects." d. "Systemic effects are intensified with inhaled doses."

Answer: D

A parent calls to ask about giving a medication for diarrhea to his child, 12 years of age, who is recovering from the flu. The nurse expects the prescriber to recommend which medication? A. bismuth subsalicylate (Pepto-Bismol) B. Lactobacillus GG (culturelle) C. belladonna alkaloid/phenobarbital combination (Donnatal Elixir) D. loperamide (Imodium A-D)

ANSWER: C → Although sucralfate does not inhibit secretion of gastric acid or pepsin and it has little neutralizing effect on gastric acid, it binds to normal and ulcerated mucosa and is used to prevent and treat peptic ulcer disease. Sucralfate can prevent absorption of a number of other medications, so other drugs with interactions should be taken 2 hours prior to a dose of sucralfate. Patients with an ulcer need to be aware that to be effective, sucralfate must be taken for 4 to 8 weeks or until healing is confirmed by radiologic or endoscopic examination.

A patient asks the nurse how sucralfate (Carafate) works. The nurse replies that sucralfate A. Inhibits gastric acid secretion B. Increases mucus and bicarbonate secretion C. Binds to normal and ulcerated mucosa, creating a protective barrier D. Enhances prostaglandin synthesis

Answer: A Rationale: The ability of a drug to be selective for receptor sites in a patient determines the types of effects it can have on the body. This drug can bind to two different types of receptors that cause different reactions. The modified occupancy theory addresses the strength of an attraction between a drug and a receptor and the drug's ability to activate the receptor. Relative potency describes the amount of drug needed to produce a specific effect. Most drugs remain bound to receptors permanently, causing the effects to be reversible.

A patient asks why albuterol causes a feeling of jitteriness when it is used to treat wheezing. The nurse knows that albuterol is a beta-adrenergic agonist that acts on beta2 receptor sites to cause smooth muscle dilation in the bronchioles of the lungs, but that it also can sometimes act on beta1 receptor sites in skeletal muscles to cause tremors. To explain this to the patient, the nurse will rely on knowledge of: a. drug selectivity. b. modified occupancy theory. c. relative potency. d. reversible effects.

Answer: C Rationale: The drug is usually taken with food to decrease gastrointestinal distress. Antacids decrease the absorption of this medication. Taking the medication on an empty stomach will not help the gastric pain. Discontinuing the medication is not recommended for this side effect. Awarded 0.0 points out of 1.0 possible points.

A patient complains of abdominal discomfort while taking nitrofurantoin. What will the nurse teach the patient? A Take the medication with an antacid. B Take the medication immediately before dinner. C Eat when taking the medication. D Discontinue the medication.

Answer: 1 Rationale: Taking the tetracycline along with an iron-containing drug such as a multivitamin may impair absorption of the tetracycline. Options 2, 3, and 4 are incorrect. Taking the tetracycline along with an iron-containing drug would not decrease, increase, or impair distribution, metabolism, or excretion of the drug. These pharmacokinetic processes occur after absorption has taken place.

A patient has an order for a tetracycline antibiotic and has been instructed to avoid taking the medication with foods, beverages, or drugs that contain calcium, iron, or magnesium. The patient takes the antibiotic along with a daily multivitamin, not realizing that the vitamin con- tains iron. What effect may this have on the tetracycline? 1. Impaired absorption 2. Increased distribution 3. Decreased metabolism 4. Impaired excretion

Answer: B

A patient has been advised to add a nasal spray (an adrenergic decongestant) to treat a cold. The nurse will include which instruction? A. "You won't see effects for at least 1 week." B. "Limit use of this spray to 3 to 5 days." C. "Continue the spray until nasal stuffiness has resolved." D. "Avoid use of this spray if a fever develops."

Answer: B Rationale: Blood disorders such as hemolytic anemia, aplastic anemia, and low white blood cell and platelet counts could result from prolonged use and high dosages. The nurse should assess the patient before assuming vitamin K deficiency, potential abuse, or frequent falls.

A patient has been on sulfonamides for urinary tract infections. The nurse assesses the patient and finds bruises on the legs and arms. What is the nurse's best action? A Ask the patient if someone is abusing her. B Assess the patient's platelet counts. C Tell the patient to be more careful. D Administer vitamin K to the patient.

Answer: B Rationale: The patient should drink 2 ounces of water after taking aluminum hydroxide to ensure the drug reaches the stomach. Aluminum hydroxide should not be taken at mealtime as it slows gastric emptying time. Aluminum hydroxide should not be taken within 1 to 2 hours of other oral medications. The patient should contact the health care provider if constipation develops as the antacid may need to be changed; self-treatment should be avoided.

A patient has been prescribed aluminum hydroxide (Amphojel) and has received patient teaching. Which statement by the patient indicates an understanding of the instructions? A "I will take aluminum hydroxide at mealtime." B "I will drink 2 ounces of water after taking aluminum hydroxide." C "I will take aluminum hydroxide within 30 minutes of my other medications." Incorrect D "I will take a laxative if I develop constipation."

Answer: D Rationale: Increased fluid intake is highly recommended to avoid complications such as crystallization in the urine. The course of therapy is not always 14 days; the patient does not have to take the drug on a full stomach, and the drug is not prescribed only for urinary tract infections.

A patient has been prescribed trimethoprim-sulfamethoxazole (Bactrim, Septra). What is the nurse's primary intervention for this patient? A Instruct the patient to take the medication for 14 days. B Ensure the patient eats something when taking the medication. C Assess the patient's urine before and after treatment. D Instruct the patient to increase fluids in the diet.

Answer: A Rationale: Phenazopyridine hydrochloride (Pyridium) is a urinary analgesic prescribed to relieve the pain associated with urinary tract infections. The other drug options are anti-infective agents and antispasmodic drugs. Having the patient take a warm bath will not address the pain.

A patient is complaining of urinary pain after being diagnosed with a urinary tract infection the previous day. What is the nurse's best action? A Administer ordered phenazopyridine hydrochloride (Pyridium). B Administer ordered trimethoprim (Trimpex). C Administer ordered bethanechol (Urecholine). D Administer ordered acetaminophen (Tylenol) and a warm bath.

Answer: C Rationale: Nystatin (Mycostatin) is an antifungal ointment that is used for a variety of candidal infections. The patient needs to be taught how to "swish and swallow" to treat this infection. There is no need to brush the teeth hourly or administer Valtrex, and starting an IV is an extreme measure.

A patient is diagnosed with an oral candidal infection. Which intervention is best? A Start an IV so the patient does not have to eat by mouth. B Instruct the patient to brush her teeth and gargle hourly. C Teach the patient how to take nystatin (Mycostatin). D Administer valacyclovir hydrochloride (Valtrex) and monitor lips and gums.

Answer: A Rationale: Antitussive medications also affect the central nervous system, thus causing drowsiness and dizziness. There is no reason to anticipate that the medication will cause diarrhea, abdominal cramping, tremors and anxiety, or headache and hypertension.

A patient is prescribed an antitussive medication. What is the most important instruction for the nurse to include in the patient teaching? A "This medication may cause drowsiness and dizziness." B "Watch for diarrhea and abdominal cramping." C "This medication may cause tremors and anxiety." D "Headache and hypertension are common side effects."

Answer: C Rationale: The magnesium saline laxatives are to be used with caution in patients with renal insufficiency, because they can be absorbed enough to cause hypermagnesemia. They are most commonly used to evacuate the bowel rapidly in preparation for endoscopic examination and to help remove unabsorbed poisons from the GI tract.

A patient is prescribed an over-the-counter laxative magnesium hydroxide (milk of magnesia). Which disorder in the patient's history would alert the nurse that there is a need to discuss this medication with the patient and the provider? A. Hypertension B. Liver dysfunction C. Renal insufficiency D. Chronic constipation

Answer: A Rationale: Co-trimoxazole increases the hypoglycemic response when taken with sulfonylureas (oral hypoglycemic agents). The nurse should assess blood sugar and determine what oral hypoglycemic the patient is taking.

A patient with type 2 diabetes mellitus is started on co-trimoxazole (TMP-SMZ). Which nursing intervention is a priority for this patient? A Assess blood sugar. B Monitor platelet count. C Assess hemoglobin and hematocrit. D Take blood pressure every 4 hours.

Answer: B Rationale: When using an anticholinergic in conjunction with an inhaled glucocorticoid or cromolyn, the ipratropium should be used 5 minutes before the steroid. This causes the bronchioles to dilate so the steroid or cromolyn can get deeper into the lungs.

A patient is prescribed ipratropium (Atrovent) and cromolyn sodium (Intal). What will the nurse teach the patient? A. Do not take these medications within 4 hours of each other." B. "Take the ipratropium (Atrovent) at least 5 minutes before the cromolyn (Intal)." C. "Administer both medications together in a metered-dose inhaler." D. "Take the ipratropium (Atrovent) only in the mornings."

Answer: B Rationale: When using an anticholinergic in conjunction with an inhaled glucocorticoid or cromolyn, the ipratropium should be used 5 minutes before the steroid. This causes the bronchioles to dilate so the steroid or cromolyn can get deeper into the lungs.

A patient is prescribed ipratropium (Atrovent) and cromolyn sodium (Intal). What will the nurse teach the patient? A "Do not take these medications within 4 hours of each other." B "Take the ipratropium (Atrovent) at least 5 minutes before the cromolyn (Intal)." C "Administer both medications together in a metered-dose inhaler." D "Take the ipratropium (Atrovent) only in the mornings."

Answer: D Rationale: Drug combination therapy is commonly used to manage chemotherapy-induced nausea and vomiting. Both lorazepam (Ativan) and the glucocorticoids have been found to be effective medications to assist in preventing and managing chemotherapy-induced nausea and vomiting.

A patient is prescribed lorazepam (Ativan) and a glucocorticoid during chemotherapy treatments. What is the nurse's best action? A Call the health care provider and question the order. B Only administer the Ativan if the patient seems anxious. C Administer the two medications at least 12 hours apart. D Administer the medications and assess the patient for relief.

Answer: D,E,F Rationale: This medication is used for motion sickness and has anticholinergic side effects, including dizziness, drowsiness, dry mouth, and constipation. The patient can use it for longer than 3 days, but must switch ears. It should be applied 4 hours before the effect is needed.

A patient is prescribed scopolamine. What information should the nurse include on the teaching plan for this patient? (Select all that apply.) A "Do not take this medication if you are dizzy." B "Do not use laxatives while on this medication." C "Do not use this medication for longer than a day." D "After 3 days, switch patch to alternate ear." E "Apply patch 4 hours before effect is desired." F "Drowsiness is a concern while on this medication."

Answer: c. notify the healthcare provider of the level

A patient is receiving IV aminophylline. The nurse checks the patient's lab values. The serum theophylline level is 32mcg/mL. What action should the nurse take? a. assess the patient's breath sounds for improvement b. increase the dosage per sliding scale directions c. notify the healthcare provider of the level d. have the laboratory collect another sample to verify the results

Answser: a) The importance of blood tests to monitor serum concentrations Rationale: The nurse should inform patients about the importance of blood tests to monitor serum concentration. The therapeutic range of theophylline is between 5 and 15 μg/mL. The patient is at risk of developing hypokalemia.

A patient is receiving theophylline (Theo-Dur) for long-term control and prevention of asthma symptoms. Patient teaching related to this medication will include which of the following? a) The importance of blood tests to monitor serum concentrations b) Taking the medication at least 1 hour prior to meals c) Development of hyperkalemia d) Monitoring liver function studies as prescribed

Answer: D Rationale: A beta agonist dilates respiratory smooth muscle, but as a side effect, it can stimulate the heart. A heart rate of 110 beats per minute is a concern, because this medication may further increase the already elevated heart rate. A pulse oximetry reading of 88% is a concern, but the medication causes bronchodilation and increased oxygenation; this should increase the pulse oximetry reading. A blood pressure of 100/60 mm Hg is on the low side, but this medication may actually cause an increase in blood pressure as a side effect; this should not concern the nurse before administration of the medication. A respiratory rate of 28 breaths per minute is elevated; however, this medication should increase oxygenation by bronchodilation, and the patient's respiratory rate should decrease once oxygenation has improved. Therefore, this should not concern the nurse.

A patient is to receive a beta agonist. Before administration of this medication, which assessment finding would most concern the nurse? a. Pulse oximetry reading of 88% b. Blood pressure of 100/60 mm Hg c. Respiratory rate of 28 breaths per minute d. Heart rate of 110 beats per minute

Answer: A Rationale: Trough levels are drawn just before infusion. Peak serum drug levels should be drawn 30 to 60 minutes after the medication is infused. The nurse should document the time drug administration is started and completed and the exact time a peak and/or trough level is drawn.

A patient on antibiotic therapy needs trough levels drawn. Which is the most appropriate time for the nurse to draw the trough level? A 10 minutes before administration of the intravenous antibiotic B 30 minutes after beginning administration of the intravenous antibiotic C 60 minutes after completion of the intravenous antibiotic infusion D 90 minutes after the intravenous antibiotic is scheduled to be administered

Answer: B Rationale: Azithromycin (Zithromax) is one of the newer macrolide antibiotics. It has a longer duration of action as well as fewer and less severe gastrointestinal side effects than erythromycin. Awarded 1.0 points out of 1.0 possible points.

A patient prescribed azithromycin (Zithromax) expresses concern regarding gastrointestinal upset experienced when taking erythromycin. What will the nurse tell this patient? A "I will call the doctor and ask for a different antibiotic." B "This drug is like erythromycin but has less severe gastrointestinal side effects." C "You need this medication and will have to tolerate the nausea." D "I will ask the doctor for a prescription for an antiemetic for possible nausea."

Answer: D

A patient receiving INH for TB reports anorexia, fatigue, malaise, jaundice, & nausea. The nurse should recognize these symptoms as indicative of A. tyramine reaction B. hepatitis C. anemia D. peripheral neuropathy

C Anticholinergic drugs block the parasympathetic nervous system, which causes the body to "rest and digest." Blocking of these effects leads to constipation, urinary retention, and decreased secretions (dry mouth).

A patient receiving an anticholinergic drug to treat nausea and vomiting should be taught to expect which side effect? A) Diarrhea B) Lacrimation C) Dry mouth D) Bradycardia

ANSWER: C → People should not use Bisacodyl frequently or for longer than 1 week, because it may produce serum electrolyte and acid—base imbalances (e.g., hypocalcemia, hypokalemia, metabolic acidosis or alkalosis). Assessment of electrolytes would thus be warranted in the care of this patient. Reflexes, heart rate, and coagulation factors may be affected, but these would be consequent to alterations in electrolyte levels.

A patient reveals to the nurse that she has been taking OTC bisacodyl on a daily basis for several months. In light of this revelation, the nurse should prioritize what assessment? A. Assessment of deep tendon reflexes B. Assessment of coagulation factors C. Assessment of electrolyte levels D. Assessment of apical heart rate

Answer: C Rationale: The patient is displaying adverse reactions to theophylline, and her blood level should be assessed before another dose of the medication. The nurse should hold the medication.

A patient taking an oral theophylline drug is due for her next dose and has a blood pressure of 100/50 mm Hg and a heart rate of 110 bpm. The patient is irritable. What is the nurse's best action? A Continue to monitor the patient. B Call the health care provider. C Hold the next dose of theophylline. D Administer oxygen 2 L per minute via nasal cannula.

Answer: A

A patient who has a history of asthma is experiencing an acute episode of shortness of breath and needs to take a medication for immediate relief. The nurse will choose which medication that is appropriate for this situation? A. a beta agonist, such as albuterol b. an leukotriene receptor antagonist, such as montelukast c. a corticosteroid, such as fluticasone d.an anticholinergic, such as ipratropium

Answer: A. a beat agonist, such as albuterol

A patient who has a history of asthma is experiencing an acute episode of shortness of breath and needs to take a medication for immediate relief. The nurse will choose which medication that is appropriate for this situation? a. a beat agonist, such as albuterol b. a leukotriene receptor antagonist, such as montelukast c. a corticosteroid, such as fluticasone d. an anticholinergic, such as ipratropium

Answer: B Rationale: Patients who are allergic to penicillins have a fourfold to sixfold increased risk of allergy to other beta-lactam antibiotics. The incidence of cross-reactivity between cephalosporins and penicillins is reported to be between 1% and 4%.

A patient who is allergic to penicillin is at increased risk for an allergy to which drug? A) tetracycline B) cefazolin sodium (Ancef) C) gentamicin (Garamycin) D) erythromycin

b. 15-60 minutes Bisacodyl suppositories usually are effective within 15 to 60 minutes of administration, so the nurse should plan accordingly to assist the patient to use the bedpan or commode.

A patient who is given a bisacodyl (Dulcolax) suppository asks the nurse how long it will take to work. The nurse replies that the patient will probably need to use the bedpan or commode within which time frame after administration? a. 2-5 minutes b. 15-60 minutes c. 2-4 hours d. 6-8 hours

Answer: C Rationale: Ethambutol (Myambutol) can cause optic neuritis. Ophthalmologic examinations should be performed periodically to assess visual acuity.

A patient with Mycobacterium tuberculosis is prescribed ethambutol (Myambutol) for long-term use. Which statement by the patient indicates understanding of the instructions? A "Dizziness, drowsiness, and decreased urinary output are common with this drug, but they will subside over time." B "Constipation will be a problem, so I will increase the fiber and fluids in my diet." C "I will need to have my eyes checked regularly while I am taking this drug." D "This medication may cause my bodily secretions to turn red-orange."

Answer: C Rationale: In an acute asthmatic attack, the short-acting sympathomimetics are the first line of defense. A beta2-adrenergic agonist will provide immediate relief, while a glucocorticoid will not; there is no need to call a code.

A patient with a history of asthma is short of breath and says, "I feel like I'm having an asthma attack." What is the nurse's highest priority action? A Calling a code B Asking the patient to describe the symptoms C Administering a beta2 adrenergic agonist D Administering a long-acting glucocorticoid

Answer: C Rationale: The nurse should instruct the patient that the urine may turn a harmless brown color. Fluids should be increased, not decreased, because this helps minimize gastrointestinal upset. Antacids should be avoided because they interfere with drug absorption. Patients with diabetes mellitus should not use Clinitest for glucose testing because a false-positive result may occur owing to changes in the urine.

A patient with a history of diabetes mellitus is prescribed nitrofurantoin (Macrodantin) to treat a urinary tract infection. Which information will the nurse include in the patient's teaching plan? A Restrict fluid intake. B Take nitrofurantoin with an antacid. C Urine may turn brown. D Use Clinitest for glucose testing.

A) Providing IV fluids and inserting a nasogastric tube A perforated peptic ulcer requires IV replacement of fluid losses and continued gastric aspiration by NG tube. Nothing is given by mouth and gastric pH testing is not a priority. Calcium gluconate is not a medication directly relevant to the patient's suspected diagnosis and parenteral nutrition is not a priority in the short term.

A patient with a history of peptic ulcer disease has presented to the emergency department with complaints of severe abdominal pain and a rigid, boardlike abdomen, prompting the health care team to suspect a perforated ulcer. Which of the following actions should the nurse anticipate? A) Providing IV fluids and inserting a nasogastric tube B) Administering oral bicarbonate and testing the patient's gastric pH level C) Performing a fecal occult blood test and administering IV calcium gluconate D) Starting parenteral nutrition and placing the patient in a high-Fowler's position

ANSWER: B → Duodenal ulcers are associated with cigarette smoking. The ulcers of smokers heal more slowly and recur more rapidly than do those of nonsmokers.

A patient with recent abdominal pain has a 40 pack-year smoking history. He reports his smoking has decreased to approximately ½ pack/day. They physician, after thorough assessment and evaluation and labs, prescribes a treatment regimen for duodenal ulcer. What important teaching would you include in relation to treatment? A. The connection between smoking and chronic pain B. Smoking's effects on the healing ulcers C. General health dangers of smoking D. The importance of taking prescribed medication until he is feeling better

Answer: A at bed time, ranitidine (Zantac) is an H2 blocker

A physician has written an order for ranitidine (Zantac) once daily. The nurse schedules the medications for which of the following times? A. At bed time B. After lunch C. With supper D. Before breakfast

Answer: B Bisacodyl (Dulcolax) is a stool softner

A physician prescribes bisacodyl (Dulcolax) for a client in preparation for a diagnostic test and wants the client to achieve a rapid effect from the medication. The nurse then tells the client to take the medication: A. With a large meal B. On an empty stomach C. At bedtime with a snack D. With two glasses of juice

Answer: D

A potential adverse effect associated with the administration of SMZ-TMP is: A. Iron deficiency anemia B. Heart failure C. Migraine headache D. crystalluria

Answer: D

A pt who has been receiving chemotherapy tells the nurse that he has been searching the internet for antinausea remedies and that he found a reference to a product called Emetrol (phosphorated carbohydrate solution). He wanted to know if this drug would help him. What is the nurses best answer? A. This may be a good remedy for you. Lets talk to your physician B. This drug is used only after other drugs have not worked C. This drug is used only to treat severe nausea and vomiting caused by chemotherapy D. This drug may not help the more severe nausea symptoms associated with chemotherapy

Answer: A

A pt who is taking oral tetracycline complains of heart burn and requests antacids. Which action by the nurse is correct? A. Give tetracycline, but delay the antacid for 1-2 hours B. Give the antacid, but delay the tetracycline for at least 4 hours C. Administer both medications together D. Explain that the antacid cannot be given while the pt is taking the tetracycline

ANSWER: B Rationale: A patient who is taking ciprofloxacin should not eat calcium-fortified juices, dairy products, iron, zinc, and cal- cium products along with antacids.

A woman is taking oral ciprofloxacin for necrotizing otitis externa. When she takes the ciprofloxacin, which of the following foods should be eliminated from her diet? A. Cranberry juice B. Calcium-enriched orange juice C. Bread D. Turkey

Answer: d) advising her to begin prophylactic therapy with isoniazid (INH).

A woman whose husband was recently diagnosed with active pulmonary tuberculosis (TB) is a tuberculin skin test converter. Management of her care would include: a) scheduling her for annual tuberculin skin testing. b) placing her in quarantine until sputum cultures are negative. c) gathering a list of persons with whom she has had recent contact. d) advising her to begin prophylactic therapy with isoniazid (INH).

Answer: C

When a patient is on aminoglycoside therapy, what symptoms or laboratory results could indicate a potentially serious toxicity? A. Fever B. WBC 7,000 mm3 C. Tinnitus & hearing loss D. Decreased creatinine levels

Answer: D Rationale: Binding to diarrhea-causing bacteria for excretion. Adsorbent antidiarrheal medications bind to diarrhea-causing bacteria to form a nonabsorbable complex, which is then excreted in the stool.

Bismuth subsalicylate (Pepto-Bismol), as an adsorbent, has which mechanism of action? A. Decreased gastrointestinal motility B. Decreased gastric secretions C. Increased fluid absorption D. Binding to diarrhea-causing bacteria for excretion

Answer: C. The most recent platelet count Rationale: Piperacillin/tazobactam (Zosyn) is an extended-spectrum penicillin used primarily for infections caused by Pseudomonas aeruginosa in immunocompromised hosts. It can cause bleeding secondary to disrupting platelet function; therefore, the platelet count may be altered. High fever, painful teeth, and mouth care do not address the patient's bleeding gums.

An immunocompromised patient who is receiving piperacillin/tazobactam (Zosyn) develops oozing and bleeding from the gums. Which additional data should the nurse determine? A. Whether the patient has a fever above 100.5°F B. Whether the patient reports any painful teeth C. The most recent platelet count D. The last time mouth care was given

Answer: C

An occupational health professional is conducting a class on risks of occupational exposure to inhalation of anthrax. Should an exposure occur, the employees are told they should receive which medication? A. Penicillin and vancomycin B. Erythromycin and vancomycin C. Ciprofloxacin and doxycycline D. Tetracycline and ampicillin

Answer: B Rationale: Almost all patients given intravenous amphotericin B (Fungizone) develop fever, chills, nausea and vomiting, and hypotension. Pretreatment with an antipyretic, antihistamine, and antiemetic can minimize or prevent these adverse reactions. There is no need to treat with IV potassium or administer insulin or IV dextrose.

Before administration of intravenous amphotericin B (Fungizone), what will the nurse do? A Set up an IV solution with potassium. B Premedicate the patient with an antipyretic, antihistamine, and antiemetic as prescribed. C Administer insulin as prescribed to prevent severe hyperglycemia. D Administer intravenous dextrose as prescribed to prevent severe hypoglycemia.

Answer: 1 Rationale: Blood Dyscrasia: A diseased state of the blood, usually one in which the blood contains permanent abnormal cellular elements.

Adverse effects of sulfonamides include which of the following? 1. Blood dyscrasias 2. Blurry vision 3. Drooling 4. Achy muscles

Answer: A

After a nebulizer treatment with the beta agonist albuterol, the patient complains of feeling a little "shaky", with slight tremors of the hands. The patient's heart rate of 88 beats/min. The nurse knows that this reaction is an a. expected adverse effect of the medication b. allergic reaction to the medication c. indication that he has received an overdose of the medication d. idiosyncratic reaction to the medication

A) Drowsiness (Although being given to this patient as an antiemetic, promethazine also has sedative and amnesic properties. For this reason, the patient is likely to experience drowsiness as an adverse effect of the medication.)

After administering a dose of promethazine (Phenergan) to a patient with nausea and vomiting, the nurse explains that which of the following may be experienced as a common temporary adverse effect of the medication? A) Drowsiness B) Reduced hearing C) Sensation of falling D) Photosensitivity

d. Metoclopramide (Reglan) Swallowed air and reduced peristalsis after surgery can result in abdominal distention and gas pains. Early ambulation helps restore peristalsis and eliminate flatus and gas pain. Medications used to reduce gas pain include metoclopramide (Reglan) or alvimopan (Entereg) to stimulate peristalsis.

After an abdominal hysterectomy, a 45-year-old woman complains of severe gas pains. Her abdomen is distended. It is most appropriate for the nurse to administer which prescribed medication? a. Morphine sulfate b. Ondansetron (Zofran) c. Acetaminophen (Tylenol) d. Metoclopramide (Reglan)

Answer: d) 6 to 12 months

After diagnosing a client with pulmonary tuberculosis, the physician tells family members that they must receive isoniazid (INH [Laniazid]) as prophylaxis against tuberculosis. The client's teenage daughter asks the nurse how long the drug must be taken. What is the usual duration of prophylactic isoniazid therapy? a) 3 to 5 days b) 1 to 3 weeks c) 2 to 4 months d) 6 to 12 months

ANS: C The nurse should clarify that nasal decongestant sprays should be used for no more than 3 days to prevent rebound vasodilation and congestion. The other responses indicate that the teaching has been effective.

After discussing management of upper respiratory infections (URI) with a patient who has acute viral rhinitis, the nurse determines that additional teaching is needed when the patient says a. "I can take acetaminophen (Tylenol) to treat discomfort." b. "I will drink lots of juices and other fluids to stay hydrated." c. "I can use my nasal decongestant spray until the congestion is all gone." d. "I will watch for changes in nasal secretions or the sputum that I cough up."

Answer: A

After the nurse administers albuterol to a 24 year old client with symptoms of acute asthma attack, which of the following outcomes is best? A. Relief of wheezing B. Respiratory rate decreased from 34 to 22 C. Dyspnea rating scale decreased from 9 to 6 D. Oxygen saturation of 82%

Answer: B

After the patient takes a dose of an inhaled corticosteroid, such as fluticasone (flovent), what is the most important action the patient needs to do next? a. Hold the breath for 60 seconds b. rinse out the mouth with water c. follow the corticosteroid with a bronchodilator inhaler, if ordered d. repeat the dose in 15 minutes if the patient feels short of breath

Answer: 2

Albuterol (Proventil HFA) two puffs and fluticasone propionate (Flovent Diskus) two puffs by metered-dose inhaler have been prescribed for a client with chronic obstructive pulmonary disease. The nurse caring for the client provides instructions regarding administration of the medication. Which statement by the client indicates an understanding of how to take these medications? 1. "I will take the two puffs of the fluticasone propionate first and then the two puffs of the albuterol." 2. "I will take the two puffs of the albuterol first and then the two puffs of the fluticasone propionate." 3."I will alternate a single puff of each, beginning with the fluticasone propionate."

Answer: A

Amoxicillin (Amoxil) is prescribed for a client who has a respiratory infection. The nurse is teaching the client about this medication and realizes that more teaching is needed when the client makes which statement? a."I should not take my medication with food." b."I will take my entire prescription of medication." c."I should report to the physician any genital itching." d."I should report to the health care provider any excess bleeding."

Answer: C

Amphotericin B, an antifungal, is: A. Fungistatic B. Fungicidal C. Either

Answer: 3

An 80- year-old client has recently been started on cimetidine (Tagamet). The nurse monitors the client for which most frequent central nervous system (CNS) side effect of this medication? 1. Tremors 2. Dizziness 3. Confusion 4. Hallucination

Answer: B Rationale: Anticholinergic bronchodilators should be used cautiously in elderly men with benign prostatic hypertrophy and in all patients with glaucoma.

An elderly male patient is prescribed ipratropium (Atrovent) for the treatment of asthma. Appropriate nursing intervention includes: A Monitoring for development of diarrhea. B Teaching the patient to report inability to urinate. C Assessing for enlarged liver. D Teaching the patient to avoid caffeine in the diet.

ANSWER: A Rationale: Patients may take the oral preparation of Cipro with food to reduce GI upset. Concurrent use of calcium is contraindicated, and the drug is normally ever 12 hours. There is no need to avoid foods like aged cheese, red wine, and dark chocolate.

An older adult with necrotizing otitis externa has been receiving IV ciprofloxacin and is now transitioning to the oral form of the drug. What health education should the nurse provide to the patient? A. "You can take this medication with food to reduce the chance that it will upset your stomach." B. "Try to avoid foods like aged cheese, red wine, and dark chocolate until you're finished with your course of antibiotics." C. "You should take this medication once per day, ideally first thing in the morning." D. "Try to increase your intake of calcium-rich foods while you're taking this drug."

ANS: D Rationale: Antihistamines are often composed of anticholinergic drugs. In older adult clients, these medications can cause or worsen urinary retention. There is no evidence that antihistamines can worsen asthma, hypotension, or kidney stones.

An older client was prescribed an antihistamine. Which medical problem may be aggravated by this medication? a. Asthma b. Hypotension c. Kidney stones d. Urinary retention

Answer: C. Look for medical alert identification.

An unknown unconscious client with an elevated temperature is ordered intravenous penicillin. What is the best action for the nurse to take? A. Administer the medication. B. Check the chart for allergies. C. Look for medical alert identification. D. Notify the nursing supervisor.

ANSWER: A & D → Aluminum-based antacids also are given to patients with chronic renal failure to decrease absorption of phosphates in food.

Antacids are alkaline substances primarily used to directly neutralize gastric acid in the stomach to prevent/treat PUD, GERD, heartburn, and GI bleed. In what other conditions might aluminum based antacids be used? Select all that apply. A. Chronic renal failure B. Hyperkalemia C. Hyporphosphatemia D. Hyperphosphatemia

ANSWER: D → Calcium compounds have a rapid onset of action but may cause hypercalcemia and hypersecretion of gastric acid (acid rebound) due to stimulation of gastrin release. If large doses are used. Consequently, calcium compounds are rarely used in peptic ulcer disease.

Antacids are alkaline substances used to directly neutralize gastric acid in the stomach. Of the commonly used compounds, which is rarely used in PUD? A. Aluminum compounds B. Simethicone compounds C. Magnesium compounds D. Calcium compounds

Answer: A

Bisacodyl has been ordered for a client who is constipated. The nurse realizes that bisacodyl: A. increases peristalsis to produce a bowel movement. B. is incompatible with alcohol consumption. C. should be avoided during pregnancy as it is teratogenic. D. may lead to paralytic ileus.

Answer: B Rationale: Albuterol may cause nervousness. The inhaled form of the drug may cause dryness and irritation of the nose and throat, not nasal congestion; insomnia, not lethargy; and hypokalemia (with high doses), not hyperkalemia. Otther adverse effects of albuterol include tremor, dizziness, headache, tachycardia, palpitations, hypertension, heartburn, nausea, vomiting and muscle cramps.

Dr. Jones prescribes albuterol sulfate (Proventil) for a patient with newly diagnose asthma. When teaching the patient about this drug, the nurse should explain that it may cause: A. Nasal congestion B. Nervousness C. Lethargy D. Hyperkalemia

Answer: C

During a routine checkup, a patient states that she is unable to take the prescribed antihistamine because of one of its most common adverse effects. The nurse suspects that which adverse effect has been bothering this patient? A. Constipation B. Abdominal cramps C. Drowsiness D. Decreased libido

Answer: C

During a teaching session for a patient who will be receiving a new prescription for the LTRA montelukast (singulair), the nurse will tell the patient that the drug has which therapeutic effect? A. improves the respiratory drive b. loosens the removes thickened secretions c. reduces inflammation in the airway d. stimulates immediate bronchodilation

c. reduces inflammation in the airway

During a teaching session for a patient who will be receiving a new prescription for the LTRA montelukast (singulair), the nurse will tell the patient that the drug has which therapeutic effect? A. improves the respiratory drive b. loosens the removes thickened secretions c. reduces inflammation in the airway d. stimulates immediate bronchodilation

Answer: B

During antitubercular therapy with isoniazid, the patient received another prescription for pyridoxine. Which statement by the nurse best explains the rationale for this second medication? a "This vitamin will help to improve your energy levels." b "This vitamin helps to prevent neurologic adverse effects." c "This vitamin works to protect your heart from toxic effects." d "This vitamin helps to reduce gastrointestinal adverse effects."

Answer: D Rationale: A beta-agonist bronchodilator stimulates the beta receptors of the sympathetic nervous system, resulting in tachycardia, bronchodilation, hyperglycemia (if severe), and alertness.

In discharge teaching, the nurse will emphasize to a patient receiving a beta-agonist bronchodilator the importance of reporting which side effect? A Hypoglycemia B Nonproductive cough C Sedation D Tachycardia

c. Relief of nausea and vomiting Metoclopramide is classified as a prokinetic and antiemetic medication. If it is effective, the patient's nausea and vomiting should resolve. Metoclopramide does not affect blood pressure, muscle tremors, or diarrhea.

Following administration of a dose of metoclopramide (Reglan) to the patient, the nurse determines that the medication has been effective when what is noted? a. Decreased blood pressure b. Absence of muscle tremors c. Relief of nausea and vomiting d. No further episodes of diarrhea

Answer: C

For the client who is crushing nitrofurantoin (Macrodantin) tablets, what should the nurse teach the client to do? a.Expect the urine to turn blue. b.Keep the urine acidic by drinking milk. c.Rinse the mouth after oral nitrofurantoin to avoid teeth staining. d.Take an antacid with oral nitrofurantoin to avoid gastrointestinal distress.

ANSWER: B → Laxatives are contraindicated in the presence of undi- agnosed abdominal pain and distention, which may indi- cate acute pathology such as an inflamed organ. In the case of appendicitis, laxatives are contraindicated because they may lead to rupture of the appendix, with contents spilling into the abdominal cavity causing life-threatening peri- tonitis. Laxatives are appropriate for the cancer patient because narcotics are often prescribed, and may lead to constipation. Laxatives are usually part of the preparation for a colonoscopy. Laxatives are often needed in patients with decreased activity such as those with Parkinson's dis- ease to stimulate bowel function.

For which of the following patients is a laxative contraindicated? A. A patient with cancer taking daily narcotics for pain control B. A patient complaining of abdominal pain and distention C. A patient scheduled for a colonoscopy D. A patient with limited mobility due to Parkinson's disease

ANSWER: A → Histamine also causes a strong stimulation of gastric acid secretion. Vagal stimulation causes release of histamine from cells in the gastric mucosa. The histamine then acts on receptors located on the parietal cells to increase production of hydrochloric acid.

Histamine is a substance found in almost every body tissue and is released in response to specific stimuli. What is an effect of histamine release? A. Stimulation of gastric acid secretion B. Pepsin synthesis C. Denaturing of gastrin D. Decrease of gastric acid secrection

ANSWER: B → Traditional antihistamines or histamine1 receptor antagonists prevent or reduce other effects of histamine but do not block histamine effects on gastric acid production.

Histamine2 receptor antagonists are used in the treatment of upper GI disorders. Why are histamine1 receptor antagonists ineffective? A. Histamine1 receptor antagonists are outdated medications of these conditions B. Histamine1 receptor antagonists do not block histamine effects on gastric acid production C. Histamine1 receptor antagonists are ineffective in presence of antacids D. Histamine1 receptor antagonists are too costly

Answer: B Rationale: Although all of the options are appropriate nursing diagnoses, fluid volume deficit is the highest priority because it has the highest associated mortality rate.

In developing a plan of care for a patient receiving an antihistamine antiemetic agent, which nursing diagnosis is of highest priority? A Knowledge deficit regarding medication administration B Fluid volume deficit related to nausea and vomiting C Risk for injury related to side effects of medication D Alteration in comfort related to nausea and vomiting

Answer: B Although all of the options are appropriate nursing diagnoses, fluid volume deficit is the highest priority because it has the highest associated mortality rate. Although a fall or injury could also prove fatal, this diagnosis is a risk; actual nursing diagnoses have priority over potential diagnoses.

In developing a plan of care for a patient receiving an antihistamine antiemetic drug, which nursing diagnosis would be of highest priority? A) Deficient knowledge regarding medication administration B) Deficient fluid volume related to nausea and vomiting C) Risk for injury related to side effects of medication D) Anxiety related to nausea and vomiting

ANSWER: D →Effective combinations include two microbials. For the antimicrobial component, two of the following drugs—amoxicillin, calrithromycin, metronidazole, or tetracycline—are used. A single antimicrobial agent is not used because of concern about emergence of drug-resistant H. pylori organisms.

In treating Helicobacter pylori infection, a physician prescribed a patient two antimicrobials. What is the rationale for using two antimicrobials? A. One antimicrobial isn't strong enough B. More than one microbe is involved in the infection. C. One antimicrobial alters the action of the PPI or H2RA D. Two are needed to prevent the emergence of drug resistant H. Pylori organisms

Answer: D Rationale: Metoclopramide works by increasing gastrointestinal motility in the small intestine, thus minimizing gastric distention and accompanying stimulation of the vomiting center.

Metoclopramide (Reglan) is useful to treat postoperative nausea and vomiting because it A) decreases mobility in the gastrointestinal tract. B) decreases chemoreceptor stimulation. C) improves the body's response to analgesia. D) promotes motility in the small intestine.

Answer: A Rationale: Increasing water absorption into the colon Milk of magnesia is a saline laxative that increases osmotic pressure to draw water into the colon

Milk of magnesia alleviates constipation by which action? A. Increasing water absorption into the colon B. Lubricating the passage of stool C. Increasing bulk within the colon D. Stimulating nerves that regulate defecation

B. Montelukast is a leukotriene receptor antagonist. By blocking leukotrienes, it blocks bronchoconstriction and inflammation. Leukotriene modifier drugs, such as montelukast, help prevent asthma attacks but are not helpful in relieving an acute attack

Montelukast is effective in relieving the inflammation and bronchoconstriction associated with acute asthmatic attacks through which principle action? A. stabilizing mast cells B. blocking leukotriene receptors C. binding to immunoglobulin (IgE) D. decreasing prostaglandin synthesis

Answer: C

Nitrofurantoin (Macrodantin), a urinary antiinfective/antiseptic is: A. Bacteriostatic B. Bactericidal C. Either

Answer: D Rationale: However, because obstruction may occur, bulk-forming laxatives should not be given to patients with difficulty in swallowing or adhesions or strictures in the GI tract, or to those who are unable or unwilling to drink adequate fluids.

Older adults who are unable or unwilling to eat an adequate diet or who are debilitated may benefit from using bulk-forming laxatives. What is an important teaching regarding intake for anyone using bulk-forming laxatives? A. Assure adequate protein intake B. Assure adequate roughage intake C. Assure adequate fat intake D. Assure adequate fluid intake

ANSWER: D → Misoprostol is a synthetic form of prostaglandin E approved for concurrent use with nonsteroidal antiinflammatory drugs (NSAIDs) to protect gastric mucosa from NSAID-induced erosion and ulceration. The most common adverse effects are diarrhea (occurs in 10% to 40% of recipients) and abdominal cramping, which may be especially difficult for older adults to tolerate. Older adults often take large doses of NSAIDs for arthritis and therefore are at risk for development of acute gastric ulcers and gastrointestinal (GI) bleeding. Thus, they may be can- didates for treatment with misoprostol; dosages may need to be decreased to reduce the chance of GI adverse effects.

One of the most common adverse effects of misoprostol that makes the drug difficult to tolerate in older adults is A. Headache B. Constipation C. Hyperphosphatemia D. Diarrhea

Answer: C Rationale: The half-life of this drug is 8 to 12 hours, and the patient should receive it twice a day. The nurse should call the health care provider to clarify this order. The medication should not be scheduled four times a day, is not expected to cause ototoxicity, and should not have an interaction with milk. Awarded 0.0 points out of 1.0 possible points.

Oral trimethoprim-sulfamethoxazole (Bactrim, Septra) is prescribed for the patient and is being administered four times a day. What is the nurse's best action? A Schedule the medication before meals and at bedtime. B Instruct the patient about potential ringing in the ears. C Call the health care provider. D Instruct the patient not to drink milk with the medication.

Answer: C Rationale: Codeine CSS II is classified as an opioid antitussive. Promethazine with dextromethorphan and benzonatate (Tessalon Perles) are both nonopioid antitussives. Levocetirizine (Xyzal) is an antihistamine.

The health care provider indicates that the patient will be ordered an opioid antitussive. Which medication does the nurse anticipate the provider will order? A Promethazine with dextromethorphan B Benzonatate (Tessalon Perles) C Codeine CSS II D Levocetirizine (Xyzal)

Answer: C

Patient teaching regarding expectorants should instruct the patient to perform which action? A. Restrict fluids to decrease mucus production. B. Take the medication once a day only, usually at bedtime. C. Increase fluid intake to decrease viscosity of secretions. D. Increase fiber and fluid intake to prevent constipation.

Answer: 3

Patients taking a penicillin should be monitored for vaginitis because: 1. It would explain the smell in their room. 2. Vaginitis is a contraindication for penicillin. 3. Vaginitis may indicate superinfection.

Answer: A,B,D

Penicillin G (Pentids) has been prescribed for a client. Which nursing intervention(s) should the nurse include for this client? (Select all that apply.) a.Collect C & S prior to first dose. b.Monitor client for mouth ulcers. c.Instruct client to limit fluid intake to 1000 mL/day. d.Have epinephrine on hand for a potential severe allergic reaction.

Answer: B

Penicillin has a cross sensitivity with which of the following drug classes? A. aminoglycosides B. cephalosporins C. macrolydes D. tetracyclines

ANS: A Antiinflammatory inhalants are taken before exercise to prevent attacks. These drugs can do nothing for the attack in progress. They are meant to be used as prophylactic therapies.

The nurse explains to the parent of a child with exercise-induced asthma that Cromolyn, an antiinflammatory drug, should be inhaled: a. before exercise to prevent attacks. b. at the initial onset of the attack. c. during the attack to relieve symptoms. d. as often as 4 times a day.

Answer: A,B,C,E (a) Wearing sunscreen and protective clothing is important to do while on this drug. Increased sensitivity to the sun can lead to severe sunburn. (b, c) Sulfamethoxazole can form crystals that precipitate in the kidney tubules. Fluid intake prevents this complication. (e) Clients should be cautioned to take all of the drug that is prescribed for them, even if their symptoms improve or disappear soon.

Situation: A 32-year-old female with a urinary tract infection reports urinary frequency, urgency, and some discomfort upon urination. Her vital signs are stable except for a temperature of 100° F. What information does a nurse provide to this client about taking her prescribed trimethoprim/sulfamethoxazole (Bactrim)? Select all that apply. a. "Be certain to wear sunscreen and protective clothing." b. "Drink at least 3 liters of fluids every day." c. "Take this drug with 8 ounces of water." d. "Try to urinate frequently to keep your bladder empty." e. "You will need to take all of this drug to get the benefits."

Answer: c. Trimethoprim/sulfamethoxazole Rationale: Guidelines indicate that a 3-day course of trimethoprim/sulfamethoxazole or fosfomycin is effective in treating uncomplicated, community-acquired UTI in women.

Situation: A 32-year-old female with a urinary tract infection reports urinary frequency, urgency, and some discomfort upon urination. Her vital signs are stable except for a temperature of 100° F. Which drug does the nurse expect the health care provider to prescribe? a. Nitrofurantoin after intercourse b. Premarin c. Trimethoprim/sulfamethoxazole d. Trimethoprim with intercourse

Answer: 2, 4 Rationale: a therapeutic drug level that is in the accepted range indicates that the drug is at a minimally effective concentration, but not at a toxic level. Because each client response to a drug is unique, the nurse should continue monitoring the client throughout the drugs use. 1,3,5 are incorrect because individualized client responses to drugs can be highly variable, adverse effects, toxicities, or even no effect may occur at levels within therapeutic range. Therapeutic effectiveness of a drug depends on many factors and the therapeutic range of a drug is the level between minimally effective and toxic levels. It is not an indicator of how effective a drug will be in treating an individual condition

THe nurse is monitoring the therapeutic drug level for a client on vancomycin (Vancocin) and notes that the level is within the acceptable range. What does this indicate to the nurse? Select all that apply. 1. The drug should cause no toxicities or adverse affects 2. The drug level is appropriate to exert therapeutic effects 3. The dose will not need to be changed for the duration of treatment 4. the nurse will need to continue monitoring because each client response to a drug is unique

Answer: 1,2

Teaching for a patient redceiving a prescription for ciprofloxacin (Cipro) should include (Select all that apply): 1. Report unusual heel, lower leg or calf pain or difficulty walking. 2. Avoid taking the medicine with milk products and antacids. 3. Limit vitamin C, both dietary and oral vitamin forms. 4. Take her pill with an antihistamine to avoid side effects.

Answer: 2

The LPN/LVN cares for a client diagnosed with methicillin-resistant Staphylococcus aureus (MRSA) infection of an abdominal wound. The physician orders vancomycin (Vancocin) 15 mg/kg q 12 h. It is MOST important for the LPN/LVN to report which of the following? 1. Decreased white blood cells (WBCs) and increased eosinophils. 2. Decreased urinary output and ototoxicity. 3. Large amount of mucoid wound drainage. 4. Delayed healing with healing by second intention.

ANS: D The client should not stop taking this drug abruptly because it suppresses the adrenal gland's production of corticosteroids. This could create a life-threatening situation. The client can expect weight gain and hyperglycemia. Use of the drug with the onset of asthma symptoms is not characteristic of Deltasone. The client should take the drug with food.

The client has recently been placed on prednisone (Deltasone). What is the highest priority instruction the nurse will provide? a. "Expect to experience weight loss and hypoglycemia." b. "Use the drug with the onset of asthma symptoms." c. "Take the drug on an empty stomach." d. "Do not stop taking the drug abruptly."

Answer: A,B,E

The nurse identifies which statements about penicillins as true? (Select all that apply.) A. Penicillins are the safest antibiotics available. B. The principal adverse effect of penicillins is allergic reaction. C. A patient who is allergic to penicillin always has a cross-allergy to cephalosporins. D. A patient who is allergic to penicillin is also allergic to vancomycin, erythromycin, and clindamycin. E. Penicillins are normally eliminated rapidly by the kidneys but can accumulate to harmful levels if renal function is severely impaired.

ANS: C Rationale: Drug therapy, including antihistamines and decongestants, is prescribed but must be used with caution because of side effects such as vertigo, hypertension, urinary retention, and insomnia. Antihistamines block the chemicals released by white blood cells from binding to receptor sites on blood vessels and nasal tissues, preventing local edema and itching. Rhinitis caused by overuse of nose drops or sprays is treated by discontinuing the offending drug. The use of vitamin C or echinacea will probably not affect acute rhinitis. The time period for contagion is not the most important part of the plan of care.

The client is experiencing acute rhinitis. Which intervention is most important for the nurse to include in this client's plan of care? a. Teach the client that he is most contagious during the 2 to 3 days prior to symptoms beginning. b. Verify that the client is taking large doses of vitamin C along with echinacea daily. c. Monitor for drug side effects such as vertigo, hypertension, urinary retention, and insomnia. d. Teach the client to use nasal sprays or drops on a frequent basis and as needed.

Answer: B Rationale: Dextromethorphan (Benylin) is classified as a non-opioid cold medication.

The client is prescribed dextromethorphan (Benylin) for treatment of a cold. The nurse teaches that the action of this drug: A. Is an opioid. B. Is a non-opioid. C. Is an anti-infective. D. Is an anti-inflammatory.

Answer: B

The client who chronically uses nonsteroidal anti-inflammatory drugs (NSAIDs) has been taking misprostol (Cytotec). The nurse determines that the medication is having the intended therapeutic effect if which of the following is noted? A. Resolved diarrhea B. Relief of epigastric pain C. Decreased platelet count D. Decreased white blood cell count

Answer: D. One hour before meals and at bedtime Rationale: Sucralfate is a gastric protectant. The medication should be scheduled for administration 1 hour before meals and at bedtime. The medication is timed to allow it to form a protective coating over the ulcer before food intake stimulates gastric acid production and mechanical irritation. The other options are incorrect.

The client with a gastric ulcer has a prescription for sucralfate (Carafate), 1 g by mouth 4 times daily. The nurse schedules the medication for which times? A. With meals and at bedtime B. Every 6 hours around the clock C. One hour after meals and at bedtime D. One hour before meals and at bedtime

Answer: B

The following antibacterial drugs have the action of inhibiting nucleic acid synthesis A. penicillin B. fluoroquinolones C. cephalosporins D. vancomycin E. amphotericin B

Answer: B, C, E

The following antibacterial drugs have the action of inhibiting protein synthesis (select all that apply) A. isoniazid B. gentamycin C. tetracycline D. metronidazole E. azithromycin

Answer: A,C,D

The following antibacterial drugs have the action of inhibition of cell wall synthesis (select all that apply) A. penicillin B. fluoroquinolones C. cephalosporins D. vancomycin E. amphotericin B

Answer: A,C,D

The following antibacterial drugs have the action of interfering with cellular metabolism (select all that apply) A. nitrofurantoin B. azithromycin C. trimethoprim D. isoniazid E. gentamicin

Answer: C Rationale: The patient's symptoms are indicative of an allergic reaction to the medication.

The health care provider has prescribed lansoprazole (Prevacid) for the patient. Within 30 minutes of receiving the first dose of the medication, the patient experiences shortness of breath and develops a rash on his skin. What does the nurse expect that the patient is experiencing? A Unexpected side effect of the medication B Toxic level of the medication C Allergic reaction to the medication D Typical side effect of the medication

Answer: D Rationale: Guaifenesin (Robitussin) is classified as an expectorant. The other drugs listed are classified as first-generation antihistamines.

The health care provider indicates that the patient will be ordered an expectorant. Which medication does the nurse anticipate the provider will order? A Brompheniramine maleate (DeCongest) B Chlorpheniramine maleate (Chlor-Trimeton) C Dexchlorpheniramine maleate (Polaramine) D Guaifenesin (Robitussin)

Answer: B Rationale: Administering the bronchodilator albuterol (Proventil) first allows the other drugs to reach deeper into the lungs as the bronchioles dilate. Anticholinergics such as ipratropium bromide (Atrovent) also help bronchodilate, but to a lesser extent. Corticosteroids such as beclomethasone (Vanceril) do not dilate and are therefore given last.

The health care provider orders ipratropium bromide (Atrovent), albuterol (Proventil), and beclomethasone (Vanceril) inhalers for a patient. What is the nurse's best action? A. Question the order; three inhalers should not be given at one time. B. Administer the albuterol, wait 5 minutes, administer ipratropium bromide, then beclomethasone several minutes later. C. Administer each inhaler at 30-minute intervals. D. Administer beclomethasone, wait 2 minutes, administer ipratropium bromide, then albuterol several minutes later.

Answer: B Rationale: Administering the bronchodilator albuterol (Proventil) first allows the other drugs to reach deeper into the lungs as the bronchioles dilate. Anticholinergics such as ipratropium bromide (Atrovent) also help bronchodilate, but to a lesser extent. Corticosteroids such as beclomethasone (Vanceril) do not dilate and are therefore given last.

The health care provider orders ipratropium bromide (Atrovent), albuterol (Proventil), and beclomethasone (Vanceril) inhalers for a patient. What is the nurse's best action? A Question the order; three inhalers should not be given at one time. B Administer the albuterol, wait 5 minutes, administer ipratropium bromide, then beclomethasone several minutes later. C Administer each inhaler at 30-minute intervals. D Administer beclomethasone, wait 2 minutes, administer ipratropium bromide, then albuterol several minutes later.

Answer: C Rationale: Lansoprazole (Prevacid) is a proton pump inhibitor that is effective in suppressing gastric acid secretions. An absence of esophageal pain would be an indication that the patient does not have reflux esophagitis.

The health care provider prescribes lansoprazole (Prevacid) for a patient. Which assessment indicates to the nurse that the medication has had a therapeutic effect? A The patient has no diarrhea. B The patient has no gastric pain. C The patient has no esophageal pain. D The patient is able to eat.

ANS: A Excessive use of steroid inhalers reduces local immune function and increases the client's risk for oral-pharyngeal infections, including candidiasis. There is no evidence that the common cold, long-term use of the same oxygen tank, or using a toothbrush for a longer period will increase the client's chances of developing candidiasis.

The home care nurse observes oral candidiasis in the client with severe, chronic, airflow limitation. What information will the nurse obtain from this client? a. "How often are you using your steroid inhaler?" b. "Have you had a cold or other viral infection lately?" c. "When was the last time the oxygen tank was changed?" d. "Have you changed your toothbrush recently?"

Answer: 3

The home health nurse performs a follow-up visit for an elderly client receiving isoniazid (INH) 200 mg every day for six months. The nurse is MOST concerned if the client makes which of the following statements? 1. "I have blurred vision at times." 2. "My legs and knees hurt." 3. "My hands and feet tingle." 4. "I think I had a migraine yesterday."

Answer: C

The nurse is caring for a patient who is receiving a high dose of intravenous azithromycin to treat an infection. The patient is also taking acetaminophen for pain. The nurse should expect to review which lab values when monitoring for this drug's side effects? a. Complete blood counts b. Electrolytes c. Liver enzymes d. Urinalysis

Answer: A

The nurse is caring for a patient who is receiving a high dose of tetracycline (Sumycin). Which laboratory values will the nurse expect to monitor while caring for this patient? a. Blood urea nitrogen (BUN) and creatinine levels b. Complete blood counts c. Electrolytes d. Liver enzyme levels

Answer: A,B,C,D,E

The nurse acknowledges which nursing intervention(s) for the client taking ciprofloxacin (Cipro)? (Select all that apply.) a.Obtain culture prior to drug administration. b.Tell the client to avoid taking Cipro with antacids. c.Monitor the client for hearing loss. d.Encourage fluids to prevent crystalluria. e.Infuse IV Cipro over 60 minutes. f.Monitor blood glucose, as Cipro can decrease effects of oral hypoglycemic.

3. Theophylline (Theo-Dur) is a bronchodilator that is administered to relax airways and decrease dyspnea. Theophylline is not used to treat infections and does not decrease or thin secretions.

The nurse administers theophylline (Theo-Dur) to a client. To evaluate the effectiveness of this medication, which of the following drug actions should the nurse anticipate? 1. Suppression of the client's respiratory infection. 2. Decrease in bronchial secretions. 3. Relaxation of bronchial smooth muscle. 4. Thinning of tenacious, purulent sputum.

ANS: C Methylxanthines, including aminophylline, stimulate the sympathetic nervous system, the cardiovascular system, and the kidneys. The development of a seizure would indicate central nervous system irritability. A bradycardiac heart rate would not be a side effect of the medication. Urinary output of 30 mL per hour is a normal assessment finding. A blood pressure of 100/60 is also considered within normal range.

The nurse assesses a client receiving aminophylline. What assessment finding indicates a dangerous side effect of this medication? a. Heart rate of 50 beats/min b. Urinary output of 30 mL in 1 hour c. Development of seizures d. Blood pressure of 100/60 mm Hg

b. Apply a cool washcloth to the forehead and provide mouth care. Rationale: Cleansing the face and hands with a cool washcloth and providing mouth care are appropriate comfort interventions for nausea and vomiting. Ginseng is not used to treat postoperative nausea and vomiting. Unnecessary activity should be avoided. The patient should rest in a quiet environment. Medications may be temporarily withheld until the acute phase is over, but the medications should not be discontinued without consultation with the health care provider.

The nurse cares for a postoperative patient who has just vomited yellow green liquid and reports nausea. Which action would be an appropriate nursing intervention? a. Offer the patient a herbal supplement such as ginseng. b. Apply a cool washcloth to the forehead and provide mouth care. c. Take the patient for a walk in the hallway to promote peristalsis. d. Discontinue any medications that may cause nausea or vomiting.

Answer: B

The nurse caring for a patient who will receive penicillin to treat an infection asks the patient about previous drug reactions. The patient reports having had a rash when taking amoxicillin (Amoxil). The nurse will contact the provider to a. discuss giving a smaller dose of penicillin. b. discuss using erythromycin (E-mycin) instead of penicillin. c. request an order for diphenhydramine (Benadryl). d. suggest that the patient receive cefuroxime (Ceftin).

C) Epigastric pain Famotidine is an H2-receptor antagonist that inhibits parietal cell output of HCl acid and minimizes damage to gastric mucosa related to hyperacidity, thus relieving epigastric pain.

The nurse determines that a patient has experienced the beneficial effects of medication therapy with famotidine (Pepcid) when which of the following symptoms is relieved? A) Nausea B) Belching C) Epigastric pain D) Difficulty swallowing

Answer: A Rationale: Albuterol is a β2-agonist that can sometimes cause adverse cardiovascular effects. These would include tachycardia and angina. A pulse rate of 72 indicates that the patient did not experience tachycardia as an adverse effect.

The nurse determines that the patient is not experiencing adverse effects of albuterol (Proventil) after noting which patient vital sign? A. Pulse rate of 72/minute B. Temperature of 98.4° F C. Oxygen saturation 96% D. Respiratory rate of 18/minute

Answer: Inhalation of cromolyn before vigorous activity

The nurse educates a patient with asthma about what action that may be effective in reducing exercise-induced asthma attacks? A. Inhalation of cromolyn before vigorous activity B. Subcutaneous injection of a beta2 agonist after strenuous activity C. Oral administration of glucocorticoids for 1 week before activity D. Transdermal application of methylxanthine before beginning activity

Answer: C Rationale: Increasing roughage (fiber) in the diet helps to add bulk to the stool. Eight glasses of water remains the recommended fluid recommendation, although there is some disagreement. Beverages with caffeine, like tea, and fatty foods like fried chicken aggravate diarrhea.

The nurse encounters a 75-year-old in the emergency department, with complaints of nausea, diarrhea, and anorexia. He has been evaluated, and it is determined that he can be treated at home. In discussing the guidelines of managing diarrhea, the nurse knows the client understands his care measures when he says: A "I will drink two glasses of water a day to prevent dehydration." B "I will drink tea when I get home." C "I will increase foods with fiber, like oatmeal." D "I will eat fried chicken for supper."

Answer: A

The nurse enters a patient's room to find that his heart rate is 120, his blood pressure is 70/50, and he has red blotching of his face and neck. Vancomycin (Vancocin) is running IVPB. The nurse believes that this patient is experiencing a severe adverse effect called "red man syndrome." What action will the nurse take? a. Stop the infusion and call the laboratory. b. Reduce the infusion to 10mg/min. c. Encourage the patient to drink more oral fluids up to 2L/day. d. Report onset of Stevens-Johnson syndrome to the health care provider.

Answer: B Rationale: Adverse effects associated with loperamide (Imodium) include central nervous system symptoms such as fatigue and dizziness, epigastric pain, abdominal cramps, nausea, dry mouth, vomiting, and anorexia. The nurse should be auscultating bowel sounds on the patient to verify that they are present in each quadrant. There is no evidence to support vascular assessment or hourly blood pressure measurements. Although intake and output is important because the patient is experiencing diarrhea, it does not have the priority that gastric assessment does.

The nurse is administering loperamide (Imodium) to a patient with diarrhea. What assessment is essential for the nurse to perform? A Vascular assessment B Gastric assessment C Hourly blood pressure measurements D Intake and output every shift

Answer: A

The nurse is administering tetracycline (Vibramycin) to a client. Which would be appropriate teaching? a.Take sunscreen precautions when at the beach. b.Take an antacid or milk with the drug to prevent severe GI distress. c.Obtain frequent hearing tests for early detection of hearing loss. d.Obtain frequent eye checkups for early detection of retinal damage.

Answer: D

The nurse is caring for a 7-year-old patient who will receive oral antibiotics. Which antibiotic order will the nurse question for this patient? a. Azithromycin (Zithromax) b. Clarithromycin (Biaxin) c. Clindamycin (Cleocin) d. Tetracycline (Sumycin)

Answer: C

The nurse is caring for a 70-kg patient who is receiving gentamicin (Garamycin) 85 mg 4 times daily. The patient reports experiencing ringing in the ears. The nurse will contact the provider to discuss a. decreasing the dose to 50 mg QID. b. giving the dose 3 times daily. c. obtaining a serum drug level. d. ordering a hearing test.

Answer: D Rationale: If a child is unable to use the inhaler, the medication will be trapped in the mouth. Using a spacer helps the medication to be deposited to the lungs.

The nurse is caring for a child who has been prescribed an inhaler for asthma control. The child is having difficulty using the inhaler. What will the nurse do? A Tell the parent to hold the inhaler for the child. B Ask the health care provider to switch to oral medications. C Tell the parent that young children should not use inhalers. D Teach the child to use a spacer.

Answer: A,B,E

The nurse is caring for a client taking nitrofurantoin (Macrodantin). Which are appropriate nursing interventions for this client? (Select all that apply.) a.Monitor urinary output and urine specific gravity. b.Monitor the client for peripheral neuropathy. c.Advise the client to wear protective clothing to prevent photosensitivity. d.Warn the client to avoid excess exposure to sunlight. e.Inform the client that urine may turn a harmless brown color

Answer: A,B,E

The nurse is caring for a client taking nitrofurantoin (Macrodantin). Which are appropriate nursing interventions for this client? (Select all that apply.) a.Monitor urinary output and urine specific gravity. b.Monitor the client for peripheral neuropathy. c.Advise the client to wear protective clothing to prevent photosensitivity. d.Warn the client to avoid excess exposure to sunlight. e.Inform the client that urine may turn a harmless brown color.

Answer: C

The nurse is caring for a client with a theophylline level of 14 mcg/mL. What is the priority nursing intervention? a. Increase the IV drip rate. b. Monitor the client for toxicity. c. Continue to assess the client's oxygenation. d. Stop the IV for an hour then restart at lower rate.

Answer: D Rationale: Second-generation antihistamines are often called non-sedating antihistamines. These may be safer for the patient to take, but the patient should still monitor for signs of excessive sedation.

The nurse is caring for a patient in the clinic who states that he is afraid of taking antihistamines because he is a truck driver. What is the best information for the nurse to give this patient? A "Take the medication only when you are not driving." B "Take a lower dose than normal when you have to drive." C "You are correct; you should not take antihistamines." D "You may be able to safely take a second-generation antihistamine."

B) Dry toast (Dry toast or crackers may alleviate the feeling of nausea and prevent further vomiting. Extremely hot or cold liquids and fatty foods are generally not well tolerated.)

The nurse is caring for a patient treated with intravenous fluid therapy for severe vomiting. As the patient recovers and begins to tolerate oral intake, the nurse understands that which of the following food choices would be most appropriate? A) Ice tea B) Dry toast C) Warm broth D) Plain hamburger

Answer: A Rationale: Acyclovir (Zovirax) is the drug of choice to treat herpes simplex infections. Ribavirin is effective against respiratory syncytial virus (RSV); zidovudine is effective against HIV, and amantadine is effective against H. influenzae type A.

The nurse is caring for a patient who has been diagnosed with genital herpes. Which medication is the drug of choice for this patient? A Acyclovir (Zovirax) B Amantadine (Symmetrel) C Ribavirin (Virazole) D Zidovudine (Retrovir)

Answer: A Rationale: Antibiotic allergy is one of the most common drug allergies. These allergies also have the potential to cause severe anaphylaxis and death and, therefore, have more importance than the other assessments listed.

The nurse is caring for a patient who has been prescribed cefazolin sodium (Ancef). Which nursing assessment is the priority? A History, including allergies B Cardiac assessment C Neurological assessment D History of immunizations

Answer: C Rationale: Tetracycline is considered to be a drug of choice for the treatment of acne rather than the other medications listed.

The nurse is caring for a patient who is being treated for acne. The nurse anticipates that the health care provider is most likely to treat the patient using which medication? A Polymyxin B Bacitracin C Tetracycline D Vancomycin

Answer: B Rationale: Misoprostol (Cytotec) is indicated for the prevention of NSAID-induced ulcer. It may be taken during NSAID therapy, including with aspirin. The patient may not be able to be completely taken off of all aspirin products; there is no evidence to support the need to take the aspirin with milk. Certain drugs like NSAIDs, which include aspirin, should be taken with food.

The nurse is caring for a patient who is experiencing gastric distress from the long-term use of aspirin for treatment of arthritis. Which intervention does the nurse anticipate that the provider may order? A Stop all aspirin therapy. B Administer misoprostol (Cytotec). C Instruct the patient to take the aspirin with milk. D Instruct the patient to take the aspirin on an empty stomach.

Answer: A Rationale: First-generation antihistamines cause drowsiness. There is no evidence to indicate that the patient should force fluids, take the medication on an empty stomach, or place the medication on hold for any period of time.

The nurse is caring for a patient who is taking a first-generation antihistamine. What is the most important information for the nurse to teach the patient? A "Do not drive after taking this medication." B "Make sure you drink a lot of liquids while on this medication." C "Take this medication on an empty stomach." D "Do not take this medication for more than 2 days."

Answer: A Rationale: Oxybutynin (Ditropan) is used to treat overactive bladder.

The nurse is caring for a patient who is taking oxybutynin (Ditropan). The nurse recognizes that the patient is most likely experiencing which condition? A Overactive bladder B Pain upon urination C Difficulty urinating D Nighttime urination

Answer: A Rationale: Red-orange discoloration of body fluids is a common side effect of rifampin (Rifadin), but it is not harmful and does not indicate infection. There is no need to call the health care provider, collect a urine culture, or start 24-hour urine collection. Awarded 0.0 points out of 1.0 possible points.

The nurse is caring for a patient who is taking rifampin (Rifadin). The patient has a heart rate of 90 beats/min, blood pressure of 100/89 mm Hg, and red-orange urine. What is the nurse's best action? A Document the findings and teach the patient. B Call the health care provider. C Collect a urine culture. D Discard the first void and start a 24-hour urine collection.

Answer: B Rationale: As sucralfate (Carafate) is not systemically absorbed, there are few adverse effects. Constipation is an adverse effect of sucralfate, so the absence of bowel sounds and a hard abdomen would require immediate action from the nurse. The sodium level listed is considered to be within the normal range; 30 mL/hr is a normal urinary output.

The nurse is caring for a patient who is taking sucralfate (Carafate) for treatment of a duodenal ulcer. Which assessment requires action by the nurse? A A sodium level 140 mEq/L B Absent bowel sounds, hard abdomen C Urinary output of 30 mL/hr D A calcium level 8.5 mg/dL

Answer: C

The nurse is caring for a patient who will begin taking doxycycline to treat an infection. The nurse should plan to give this medication a. 1 hour before or 2 hours after a meal. b. with an antacid to minimize GI irritation. c. with food to improve absorption. d. with small sips of water.

Answer: C, D, E Rationale: a&b are first line drugs.

The nurse is caring for a patient with Tuberculosis, the nurse understands that second line medications include: Select all that apply: a. ethambutol HCL b. Isoniazid c. aminosalicyclate sodium d. capreomycin e. ethionamide

Answer: C

The nurse is caring for a patient with a theophylline level of 14 mcg/mL. What is the priority nursing intervention? A Increase the IV drip rate. B Monitor the patient for toxicity. C Continue to assess the patient's oxygenation. D Stop the IV for an hour then restart at lower rate.

Answer: C Rationale: The therapeutic theopylline level is 10 to 20 mcg/mL. The nurse should continue interventions and monitor oxygenation.

The nurse is caring for a patient with a theophylline level of 14 mcg/mL. What is the priority nursing intervention? A Increase the IV drip rate. B Monitor the patient for toxicity. C Continue to assess the patient's oxygenation. D Stop the IV for an hour then restart at lower rate.

Answer: D Rationale: The side effects of epinephrine include tachycardia, dysrhythmias, and palpitations. This patient should not receive epinephrine.

The nurse is caring for multiple patients on the pulmonary unit. The nurse would question the administration of prescribed epinephrine to which patient? A The patient with a history of emphysema B The patient with a history of type 2 diabetes C The patient who is 16 years old D The patient with atrial fibrillation with a rate of 100

Answer: C Rationale: The risk of ototoxicity with vancomycin is increased for patients taking furosemide. The nurse should assess this patient first. A patient who has started antibiotic therapy would be expected to have a fever. Gastric upset is common with this antibiotic. Telithromycin is recommended for patients 18 years and older.

The nurse is caring for multiple patients receiving antibiotics. Which patient will the nurse assess first? A The patient who just started azithromycin therapy with a fever B The patient who is taking clindamycin (Cleocin) and has gastric upset C The patient who is taking vancomycin (Vancocin) with furosemide (Lasix) D The patient who is taking telithromycin (Ketek) and is 18 years old

Answer: A,B,C,D

The nurse is discussing management of symptoms of an upper respiratory tract infection. Which classes of medications are often used in treating the symptoms of upper respiratory tract infections? (Select all that apply.) A.Antihistamines B. Nasal decongestants C. Antitussives D. Expectorants E. Antibiotics

Answer: A

The nurse is discussing use of antitussive medications with a patient. What common side effect does the nurse include in the patient teaching? A.Drowsiness and dizziness B. Diarrhea and abdominal cramping C. Tremors and anxiety D. Bradycardia and increased lacrimation

It should be administered with food to diminish the risk of gastric irritation.

The nurse is discussing with a patient the time of day for taking prednisone. What information would the nurse include in the teaching based on knowledge of glucocorticoids?

Answer: D Rationale: The medication is designed to be taken once over a 24-hour period, but it should be taken at the same time every day for maximum effectiveness.

The nurse is instructing the patient on the proper way to take tolterodine tartrate (Detrol, Detrol LA). The nurse instructs the patient to take the medication A three times a day at meal time on schedule. B twice a day on schedule on an empty stomach. C twice a day on schedule with a meal or a snack. D once a day at the same time every day, with or without food.

Answer: B,C,F

The nurse is monitoring for liver toxicity in a patient who has been receiving long-term isoniazid therapy. Manifestations of liver toxicity include: (Select all that apply.) a Orange discoloration of sweat and tears b Darkened urine c Dizziness d Fatigue e Visual disturbances f Jaundice

Answer: B

The nurse should instruct the patient taking nitrofurantoin and complaining of nausea and diarrhea to do what to decrease this side effect? A. Take the medication at bedtime. B. Take the medication with meals or milk. C. Take the medication with a large glass of water. D. Take the medication with bismuth subsalicylate.

d. Increases peristalsis by stimulating nerves in the colon wall Bisacodyl is a stimulant laxative that aids in producing a bowel movement by irritating the colon wall and stimulating enteric nerves. It is available in oral and suppository forms. Fiber and bulk forming drugs increase bulk in the stool; water and stool softeners soften feces, and saline and osmotic solutions cause fluid retention in the intestinal tract.

The nurse is preparing to administer a dose of bisacodyl (Dulcolax). In explaining the medication to the patient, the nurse would explain that it acts in what way? a. Increases bulk in the stool b. Lubricates the intestinal tract to soften feces c. Increases fluid retention in the intestinal tract d. Increases peristalsis by stimulating nerves in the colon wall

d. Chart the dose as not given on the medical record and explain in the nursing progress notes. Whenever a patient refuses medication, the dose should be charted as not given with an explanation of the reason documented in the nursing progress notes. In this instance, the refusal indicates good judgment by the patient, and the patient should not be encouraged to take it today.

The nurse is preparing to administer a scheduled dose of docusate sodium (Colace) when the patient reports an episode of loose stool and does not want to take the medication. What is the appropriate action by the nurse? a. Write an incident report about this untoward event. b. Attempt to have the family convince the patient to take the ordered dose. c. Withhold the medication at this time and try to administer it later in the day. d. Chart the dose as not given on the medical record and explain in the nursing progress notes.

Answer: D *Gentomicin is nephrotoxic and ototoxic. Monitor urine output, BUN/Creat.

The nurse is preparing to administer intravenous gentamicin to an infant through an intermittent needle. The nurse notes that the infant has not had a wet diaper for several hours. The nurse will perform which action? a. Administer the medication and give the infant extra oral fluids. b. Contact the provider to request adding intravenous fluids when giving the medication. c. Give the medication and obtain a serum peak drug level 45 minutes after the dose. d. Hold the dose and contact the provider to request a serum trough drug level.

Answer: A Rationale: Penicillin G is the drug of choice for treating syphilis. Miconazole is used to treat candidiasis. Metronidazole is used to treat trichomoniasis. Doxycycline is used to treat chlamydia.

The nurse is preparing to administer medication therapy to a woman diagnosed with syphilis. The nurse would expect to administer: a) Penicillin G b) Miconazole c) Doxycycline d) Metronidazole

Answer: B

The nurse is providing pt teaching regarding scopolamine transdermal patches (Transderm scop) to a pt who is planning on ocean cruise. Which instruction is most appropriate? A. Apply patch the day before traveling B. Apply the patch at least 4 hours before traveling C. Apply the patch to the shoulder area D. Apply the patch to the temple just above the ear

Answer: C

The nurse is reviewing a patient's chart prior to administering gentamycin (Garamycin) and notes that the last serum peak drug level was 9 mcg/mL and the last trough level was 2 mcg/mL. What action will the nurse take? a. Administer the next dose as ordered. b. Obtain repeat peak and trough levels before giving the next dose. c. Report possible drug toxicity to the patient's provider. d. Report a decreased drug therapeutic level to the patient's provider.

Answer: D

The nurse is reviewing the use of antitussive drugs. Antitussive drugs would be most appropriate for which patient? A. A patient who has pneumonia with a productive cough B. A patient who has a tracheostomy and thick mucus secretions C. A patient who has had a productive cough for 2 weeks D. A patient who has developed bronchitis 2 days after abdominal surgery

Answer: C

The nurse is teaching a client about sulfadiazine (Microsulfon). Which directive should the nurse include in the teaching? a.Avoid caffeine during sulfonamide treatment. b.Administer in 50 mL of fluid over 30 minutes. c.Avoid sulfonamides during the third trimester of pregnancy. d.Use an ultraviolet light to enhance drug effectiveness.

Answer: D Rationale: The syrup base of dextromethorphan will help to soothe throat irritation and fluids should be avoided immediately following administration

The nurse is teaching a client about the use of dextromethorphan with guaifensesin (Robitussin-DM) syrup for a cough accompanied by thick mucus. Which instruction should be indicated in the client's teaching? a. "Lie supine for 30 minutes after taking the liquid" b. "Drunk minimal fluids to avoid stimulating the cough reflex" c. "Take the drug with food for best results" d. "Avoid drinking fluids immediately after the syrup but increase overall fluid intake throughout the day"

Answer: C Rationale: Expectorant drugs are used to decrease viscosity of secretions and allow them to be more easily expectorated. Increasing fluid intake helps this action.

The nurse is teaching a patient about the use of an expectorant. What is the most important instruction for the nurse to include in the patient teaching? A "Restrict your fluids in order to decrease mucus production." B "Take the medication once a day only, at bedtime." C "Increase your fluid intake in order to decrease viscosity of secretions." D "Increase your fiber and fluid intake to prevent constipation."

Answer: B Rationale: It is important to breathe out slowly before positioning the inhaler. This allows the patient to take a deeper breath while inhaling the medication, thus enhancing the effectiveness of the dose. The inhaler should be shaken well. A spacer may be used. Holding the breath after the inhalation of medication helps keep the medication in the lungs, but 30 seconds will not be possible for a patient with COPD.

The nurse is teaching a patient how to self-administer ipratropium (Atrovent) via a metered dose inhaler (MDI). Which instruction given by the nurse is most appropriate to help the patient learn the proper inhalation technique? A. "Avoid shaking the inhaler before use." B. "Breathe out slowly before positioning the inhaler." C. "Using a spacer should be avoided for this type of medication." D. "After taking a puff, hold the breath for 30 seconds before exhaling."

Rationale: Tachycardia Albuterol is a bronchodilator. Side effects can include tachycardia, hypertension, chest pain, dysrhythmias, nervousness, restlessness, and headache, among others. The nurse monitors for these effects during therapy. The items in the other options are not side effects of this medication.

The nurse should monitor the client receiving the first dose of albuterol (Proventil HFA) for which side effect of this medication?

Answer: B,C,D Rationale: The medication should not be taken with an antacid, because they interfere with drug absorption. The medication should be shaken well before drinking, and the patient should increase fluids to help with nausea. This medication can stain the teeth, so swishing is not recommended. The medication can cause stomach upset and should be taken with food.

The nurse is teaching a patient who has been prescribed nitrofurantoin. The teaching plan for this patient will include which interventions? (Select all that apply.) A Swish and swallow this medication. B Do not take the medication with an antacid. C Shake the suspension well before drinking. D Increase fluids while on the medication. E Take medication on an empty stomach.

Answer: A,B,C,D

The nurse is teaching the client about trimethoprim-sulfamethoxazole (Bactrim). Which directives should be included in the teaching? (Select all that apply.) a.Report any bruising or bleeding immediately. b.Report any diarrhea or bloody stools promptly. c.Report any fever, rash, or sore throat promptly. d.Avoid unprotected exposure to sunlight. e.Report thirst and polyuria immediately.

Answer: B Rationale: Beclomethasone diproprionate (Beconase) is a steroid spray administered nasally. It is used to prevent allergy symptoms. Its effect is localized, and therefore the patient does not have systemic side effects with normal use and does not have to worry about weaning off the medication as with oral corticosteroids. Because the medication has a localized effect, it will not produce the changes in blood sugar that would be generated by systemic steroids.

The nurse is teaching the patient on the use of beclomethasone diproprionate (Beconase). Which statement by the patient indicates an understanding of the teaching? A "I will need to taper off the medication to prevent acute adrenal crisis." B "This medication will help prevent the inflammatory response of my allergies." C "I will need to monitor my blood sugar more closely because it may increase." D "I need to take this medication only when my symptoms get bad."

A. Rationale: A serum theophylline level of 25 mcg/mL is at a toxic level. The nurse should hold the dose and notify the health care provider. Serious adverse effects frequently occur at this serum drug level, including anorexia, nausea, vomiting, agitation, nervousness, insomnia, tachycardia and other dysrhythmias, and tonic-clonic convulsions. Ventricular dysrhythmias or convulsions may be the first sign of toxicity.

The nurse notes that a patient's serum theophyillne level is 25 mcg/mL and that a schedule dose of the medication is due. The nurse should A. Hold the scheduled dose, contact the health care provider, and asses the patient for signs of theophylline toxicity. B. Administer the dose as scheduled C. Administer only half the dose D. Hold the dose until the next meal and administer at that time

Answer: D Diphenhydramine should be administered undiluted at a rate of 25 mg/min.

The nurse plans to administer 50 mg diphenhydramine (Benadryl) intravenously A) diluted in 50 mL normal saline over 30 minutes. B) undiluted over 1 minute. C) diluted in 100 mL D5W over 20 minutes. D) undiluted over 2 minutes.

Answer: A

The nurse provides home care instructions for a patient who will take a high dose of azithromycin after discharge from the hospital. Which statement by the patient indicates understanding of the teaching? a. "I may take antacids 2 hours before taking this drug." b. "I should take acetaminophen for fever or mild pain." c. "I should expect diarrhea to be a common, mild side effect." d. "I should avoid dairy products while taking this drug."

c. Administer the medication deep into the muscle to prevent tissue damage. Rationale: Promethazine (Phenergan) is an antihistamine administered to relieve nausea and vomiting. Deep muscle injection is the preferred route of injection administration. This medication should not be administered into an artery or under the skin because of the risk of severe tissue injury, including gangrene. When administered IV, a risk factor is that it can leach out from the vein and cause serious damage to surrounding tissue.

The nurse receives an order for a parenteral dose of promethazine (Phenergan) and prepares to administer the medication to a 38-year-old male patient with nausea and repeated vomiting. Which action is most important for the nurse to take? a. Administer the medication subcutaneously for fast absorption. b. Administer the medication into an arterial line to prevent extravasation. c. Administer the medication deep into the muscle to prevent tissue damage. d. Administer the medication with 0.5 mL of lidocaine to decrease injection pain.

Answer: C Rationale: Aminoglycoside antibiotics, including gentamicin, have a high risk for nephrotoxicity and ototoxicity

The nurse should assess a patient for nephrotoxicity and ototoxicity when administering which antimicrobial? A. cefazolin B. clindamycin C. gentamicin D. erythromycin

Answer: A Rationale: Tetracycline is contraindicated in children younger than 8 years because it can cause permanent discoloration of the teeth. Tetracycline is not contraindicated for patients diagnosed with diabetes mellitus or hypertension. Tetracycline is used to treat rickettsiae.

The nurse should question the prescription of tetracycline for which patient? A A 6-year old patient with Haemophilus influenzae B A 45-year-old patient with a history of diabetes mellitus C A 60-year-old patient with a history of hypertension D A 40-year-old patient diagnosed with rickettsiae

Answer: A

The nurse teaches the client taking amphotericin B to report which signs and symptoms to the health care provider? a.Blindness b.Loss of hearing c.Nephrotoxicity d.Stevens-Johnson syndrome

Answer: D

The nurse understands that which antibiotic when given with penicillin will reduce the bacterial effect of the penicillin? A. Aminoglycosides B. Vancomycin C. Probenecid D. Tetracycline

Answer: C Rationale: Antileukotriene agents block the inflammatory response of leukotrienes and thus the trigger for asthma attacks. Response to these drugs is usually noticed within 1 week. They are not used to treat an acute asthma attack. Awarded 0.0 points out of 1.0 possible points.

The nurse will include which information regarding the use of antileukotriene agents such as zafirlukast (Accolate) in the patient teaching? A "Take the medication as soon as you begin wheezing." B "It will take about 3 weeks before you notice a therapeutic effect." C "This medication will prevent the inflammation that causes your asthma attack." D "Increase fiber and fluid in your diet to prevent the side effect of constipation."

C Dronabinol is the active ingredient of marijuana. It has been approved by the FDA as a second-line drug to treat nausea and vomiting related to cancer chemotherapy.

The nurse would anticipate administering which medication to a patient suffering with nausea and vomiting not relieved by first-line antiemetic drugs? A) prochlorperazine (Compazine) B) metoclopramide (Reglan) C) dronabinol (Marinol) D) phosphorated carbohydrate solution (Emetrol)

A Scopolamine blocks the binding of acetylcholine with cholinergic receptors in the inner ear, an imbalance of which is a common cause of motion sickness.

The nurse would expect to teach a patient about which antiemetic commonly used to prevent motion sickness? A) scopolamine (Transderm-Scop) B) Ddoperidol (Inapsine) C) prochlorperazine (Compazine) D) metoclopramide (Reglan)

Answer: D. Blood urea nitrogen (BUN) and creatinine Rationale: Gentamycin has a high potential for nephrotoxicity and is thus contraindicated in patients with elevated renal function tests such as BUN and creatinine

The nurse would monitor which laboratory values in a patient receiving intravenous gentamicin (Garamycin)? A. Hematocrit and hemoglobin B. Serum glutamic-oxaloacetic transaminase (SGOT) and alanine transaminase (ALT) C. Prothrombin time (PT) and partial thromboplastin time (PTT) D. Blood urea nitrogen (BUN) and creatinine

d. Magnesium hydroxide (Milk of Magnesia) Milk of Magnesia may cause hypermagnesemia in patients with renal insufficiency. The nurse should question this order with the health care provider. Bisacodyl, lubiprostone, and cascara sagrada are safe to use in patients with renal insufficiency as long as the patient is not currently dehydrated.

The nurse would question the use of which cathartic agent in a patient with renal insufficiency? a. Bisacodyl (Dulcolax) b. Lubiprostone (Amitiza) c. Cascara sagrada (Senekot) d. Magnesium hydroxide (Milk of Magnesia)

Answer: B Rationale: Corticosteroids such as prednisone can lead to weight gain. For this reason, it is important to monitor the patient's daily weight. The drug should not affect the apical pulse, bowel sounds, or deep tendon reflexes.

The nurse, who has administered a first dose of oral prednisone to a patient with asthma, writes on the care plan to begin monitoring for which patient parameters? A Apical pulse B Daily weight C Bowel sounds D Deep tendon reflexes

Answer: C

The patient asks how stool softeners relieve constipation. Which of the following would be the best response by the nurse? Stool softeners relieve constipation by: a. stimulating the walls of the intestine b. promoting the retention of sodium in the fecal mass c. promoting the retention of water in the fecal mass d. lubricating the intestinal walls

Answer: C Rationale: The drug of choice for treatment of Legionnaires' disease is erythromycin (E-Mycin).

The patient has been diagnosed with Legionnaires' disease. Which drug does the nurse anticipate the provider will order? A Daptomycin (Cubicin) B Lincomycin (Lincocin) C Erythromycin (E-Mycin) D Azithromycin (Zithromax)

Answer: C Rationale: Miconazole nitrate (Monistat, Micatin) can be ordered to treat candidiasis. The other drugs listed can be used to treat tinea pedis, corporis, and cruris.

The patient has been diagnosed with candidiasis. The nurse recognizes that the patient is most likely to be ordered which drug? A Sulconazole (Exelderm) B Haloprogin (Halotex) C Miconazole nitrate (Monistat, Micatin) D Tolnaftate (Aftate)

Answer: D Rationale: Of the drugs listed, phenazopyridine HCl (Pyridium) is known to turn the urine an orange color.

The patient has been ordered a regimen to treat a urinary tract infection and notifies the nurse that the patient's urine has turned an orange color. The nurse recognizes that the patient is most likely being treated with which drug? A Ciprofloxacin (Cipro) B Bethanechol Cl (Urecholine) C Darifenacin hydrobromide (Enablex) D Phenazopyridine HCl (Pyridium)

This drug has a longer duration of action than some of the other antibiotics

The patient has been ordered azithromycin (Zithromax) and asks the nurse why the medication does not have to be taken as often as other antibiotics that have previously been ordered. What is the nurse's best response?

Answer: B Rationale: Azithromycin (Zithromax) is one of the newer macrolide antibiotics. It has a longer duration of action as well as fewer and less severe gastrointestinal side effects than erythromycin.

The patient has been ordered azithromycin (Zithromax) and asks the nurse why the medication does not have to be taken as often as other antibiotics that have previously been ordered. What is the nurse's best response? A "I'll call the pharmacy and ask about the chemical makeup of the drug." B "This drug has a longer duration of action than some of the other antibiotics." C "You'll need to ask your health care provider questions like that." D "This is a much more effective drug than what you received previously."

Answer: D Rationale: The interaction of tetracycline with oral contraceptives tends to reduce the effectiveness of the oral contraceptives; therefore, women of childbearing years should be cautioned to utilize a secondary means of birth control.

The patient has been ordered tetracycline. She tells the nurse that she is currently taking birth control pills. The nurse should anticipate that the interaction of the tetracycline and the oral contraceptives will most likely result in A enhanced action of the tetracycline. B decreased action of the tetracycline. C enhanced action of the oral contraceptives. D decreased action of the oral contraceptives.

Answer: A Rationale: Of the instructions provided, the only one that would affect the prevention of kidney stones is to take the medication with a full glass of fluid. Awarded 1.0 points out of 1.0 possible points.

The patient has been prescribed trimethoprim-sulfamethoxazole (Bactrim, Septra). The nurse notes that the patient has a history of kidney stones. What is the highest priority instruction for the nurse to give to the patient? A "Take the medication with a full glass of fluid." B "Take the medication at regularly spaced intervals." C "Take the medication on a full stomach." D "Take the medication on an empty stomach."

Answer: C Rationale: Administering the medication with food or milk may minimize the existence of abdominal cramping. The other interventions may be helpful but will not necessarily prevent abdominal cramping.

The patient has been prescribed trimethoprim-sulfamethoxazole (Bactrim, Septra). The patient complains of experiencing abdominal cramping. What is the highest priority instruction for the nurse to give to the patient? A "Take the medication with a full glass of fluid." B "Take the medication at regularly spaced intervals." C "Take the medication with food or milk." D "Take the medication on an empty stomach."

Answer: B Rationale: The health care provider should be contacted regarding the ordering of Bactrim for this patient since it has not been shown to be effective in treating viral infections.

The patient has been started on a medication regimen that includes trimethoprim-sulfamethoxazole (Bactrim). The nurse notes that the source of the patient's infection has been determined to be viral in origin. What is the nurse's highest priority action? A Administer the medication as ordered by the provider. B Contact the health care provider to discuss the medication. C Ask the patient if he knows how he contacted the infection. D Ensure that the information is documented in the chart.

Answer: D Rationale: Amphotericin B is considered highly toxic and can cause nephrotoxicity and electrolyte imbalance, especially hypokalemia and hypomagnesemia (low serum potassium and magnesium levels). Urinary output, blood urea nitrogen, and serum creatinine levels need to be closely monitored. Awarded 0.0 points out of 1.0 possible points.

The patient is being treated with intravenous amphotericin B. What is the nurse's primary intervention? A Encourage patient to drink at least a liter of fluid per shift. B Assess the IV site for infiltration. C Administer with dextrose. D Assess blood urea nitrogen and creatinine.

Answer: D Rationale: Cromolyn (Intal) is classified as a mast cell stabilizer, since its action serves to inhibit mast cells from releasing histamine and other chemical mediators of inflammation. This drug should be taken on a daily basis, and it is not effective at terminating acute asthma attacks.

The patient is prescribed cromolyn (Intal) for treatment of asthma. The nurse teaches that the action of this drug is _________________. A Bronchodilation. B Anti-infective. C Mucolytic. D Anti-inflammatory.

Answer: D Rationale: The methylxanthines comprise a group of bronchodilators chemically related to caffeine. Because of the drugs' chemical similarities, patients should avoid foods and beverages containing caffeine when taking these drugs.

The patient is receiving theophylline (Theo-Dur) for treatment of asthma. Nursing intervention is required if the patient makes which of the following statements? A "I will notify my doctor if my vision changes." B "I will use my inhaler if I am wheezing." C "I will check my heart rate each day." D "I will take my medicine with my coffee each morning."

Answer: A Rationale: Nonselective adrenergic agonist bronchodilators stimulate beta1 receptors in the heart and beta2 receptors in the lungs. Stimulation of beta1 receptors can increase heart rate and contractility, increasing oxygen demand. This increased oxygen demand may lead to angina or myocardial ischemia in patient with coronary artery disease. Cautious use of these agents is indicated if the patient has coronary artery disease.

The patient is taking a nonselective adrenergic agonist bronchodilator and has a history of coronary artery disease. What is a priority nursing intervention? A Monitoring patient for potential chest pain B Monitoring blood pressure continuously C Assessing daily for hyperkalemia D Assessing 12-lead ECG each shift

Answer: D

The patient is taking ciprofloxacin and zinc salts. How should the patient be taught to take the medication? A. Take both medications with meals. B. Take the ciprofloxacin 30 minutes after taking the zinc salts. C. Take the ciprofloxacin 1 hour before or 2 hours after taking the zinc. D. Take the zinc salts 6 hours before the ciprofloxacin or 2 hours after the medication.

C) Ondansetron (Zofran) (Ondansetron is a 5-HT3 receptor antagonist antiemetic that is especially effective in reducing cancer chemotherapy-induced nausea and vomiting.)

The patient receiving chemotherapy rings the call bell and reports an onset of nausea. The nurse should prepare a prn dose of which of the following medications? A) Morphine sulfate B) Zolpidem (Ambien) C) Ondansetron (Zofran) D) Dexamethasone (Decadron)

Answer: C Rationale: Between 6 months and 1 year is sufficient time for prevention of active tuberculosis. Because the tuberculosis mycobacterium is slow-growing, shorter lengths of time may not sufficiently eradicate the organism.

The patient states that she has been prescribed prophylactic medication for tuberculosis for a period of 4 weeks. What is the nurse's best response? A "Let me teach you about the medications." B "We do not use medications prophylactically for tuberculosis." C "You should be on the drugs for at least 6 months." D "You should be on the medications for only 2 weeks."

Answer: C Rationale: Gentamicin (Garamycin) has a high potential for nephrotoxicity and is thus contraindicated in patients with elevated renal function tests such as BUN and creatinine. The nurse should hold the medication and call the health care provider. Increasing fluids will not decrease the patient's BUN.

The patient taking intravenous gentamicin (Garamycin) has elevated blood urea nitrogen (BUN). What is the nurse's best course of action? A Have the patient increase fluid intake. B Monitor peak and trough levels. C Hold the medication. D Insert a Foley catheter.

Answer: B Rationale: The patient needs an expectorant. This medication will help the patient cough the fluid out of her lungs. Dextromethorphan and fluticasone will not help the patient expectorate. There is no information about the patient's fluid intake, so hourly fluids may be too much. Awarded 0.0 points out of 1.0 possible points.

The patient tells the nurse that she has a cold, is coughing, and feels like she has fluid in her lungs. What action will the nurse anticipate performing next? A Administer dextromethorphan. B Administer guaifenesin. C Encourage the patient to drink fluids hourly. D Administer fluticasone (Flonase).

Answer: A Rationale: This UTI is a complicated UTI because the patient has type 2 diabetes and the UTI is recurrent. Ciprofloxacin (Cipro) would be used for a complicated UTI. Fosfomycin (Monurol), nitrofurantoin (Macrodantin), and trimethoprim/sulfamethoxazole (Bactrim) should be used for uncomplicated UTIs.

The patient with type 2 diabetes has a second UTI within one month of being treated for a previous UTI. Which medication should the nurse expect to teach the patient about taking for this infection? A. Ciprofloxacin (Cipro) Correct B. Fosfomycin (Monurol) C. Nitrofurantoin (Macrodantin) D. Trimethoprim/sulfamethoxazole (Bactrim)

Answer: A Rationale: In gram-positive cocci, gentamicin is usually given in combination with a penicillin. The other antibiotics are not typically prescribed with gentamicin for this culture result.

The patient's culture has grown gram-positive cocci, and the patient is prescribed two different antibiotics, one of which is gentamicin. To treat this type of infection, which type of drug is typically prescribed with gentamicin? A. A penicillin B. A cephalosporin C. A fluoroquinolone D. An aminoglycoside

Answer: B Rationale: The drug rabeprazole (Aciphex) is classified as a proton pump inhibitor.

The patient's health care provider prescribes rabeprazole (Aciphex) to a patient. The nurse recognizes that this drug is effective for the patient because it belongs to which drug class? A Antiinfective agent B Proton pump inhibitor C Antacid D Histamine2 blocker

Answer: d) Daily doses of isoniazid, 300 mg for 6 months to 1 year

The physician determines that a client has been exposed to someone with tuberculosis. The nurse expects the physician to order which of the following? a) Daily oral doses of isoniazid (Nydrazid) and rifampin (Rifadin) for 6 months to 2 years b) Isolation until 24 hours after antitubercular therapy begins c) Nothing, until signs of active disease arise d) Daily doses of isoniazid, 300 mg for 6 months to 1 year

Answer: C,B,A

The physician orders ipratropium bromide (Atrovent), albuterol (Proventil), and beclomethasone (Beclovent) inhalers for your patient. Put the drugs in the correct order in which they will be administered, using the choices A through C listed below. A. beclomethasone (Beclovent) B. ipratropium bromide (Atrovent) C. albuterol (Proventil)

Answer: 2. nervousness

The physician prescribes albuterol (Proventil) for a client with newly diagnosed asthma. When teaching the client about this drug, the nurse should explain that it may cause:

Answer: A Rationale: Docusate sodium (Colace) is a stool softner, retaining water in the stool, softens the mass and makes stool easier and less painful to pass.

The physician prescribes docusate sodium (Colace) for the client. The client asks the nurse to explain why the medication is needed. Which explanation given by the nurse correctly states the purpose of medication? A. To ease bowel evacuation and its related discomfort B. To irriatate the bowel and promote stool elimination C. To stimulate peristalsis to remove wastes after the digestion D. To reduce intestinal activity and decrease stool size

B) "It would likely be beneficial for you to eliminate drinking alcohol." Although there is no specific recommended dietary modification for PUD, most patients find it necessary to make some sort of dietary modifications to minimize symptoms. Milk may exacerbate PUD and alcohol is best avoided because it can delay healing.

The results of a patient's recent endoscopy indicate the presence of peptic ulcer disease (PUD). Which of the following teaching points should the nurse provide to the patient in light of his new diagnosis? A) "You'll need to drink at least two to three glasses of milk daily." B) "It would likely be beneficial for you to eliminate drinking alcohol." C) "Many people find that a minced or pureed diet eases their symptoms of PUD." D) "Your medications should allow you to maintain your present diet while minimizing symptoms."

Answer: 3 Rationale: INH is the drug of choice for chemoprophylaxis. All of the other drugs listed can be used in the treatment of tuberculosis

The wife of a client with active tuberculosis has a positive skin test for tuberculosis. She is to be started on prophylactic drug therapy. What drug is the drug of choice for prophylaxis of tuberculosis? 1. Streptomycin 2. Para-aminosalicylic (PAS) acid 3. Isoniazid (INH) 4. Ethambutol (Myambutol)

Answer: D Rationale: isoniazid competes for the available vitamin B6 in the body and leaves the client at risk for developing neuropathies related to vitamin deficiency. Supplemental vitamin B6 is routinely prescribed to address this issue. Avoiding sun exposure is a preventative measure to lower the risk of skin cancer. Following a low-cholesterol diet lowers the individual's risk of developing atherosclerotic plaque. Rest is important in maintaining homeostasis but has no real impact on neuropathies.

To prevent development of peripheral neuropathies associated with isoniazid administration, the nurse should teach the client to: a. avoid excessive sun exposure b. follow a low-cholesterol diet c. obtain extra rest d. supplement the diet with pyridoxine (vitamin B6)

Answer: B,C,D

What OTC medications should the nurse teach a patient taking ciprofloxacin to avoid? (Select all that apply.) A. Famotidine B. Aluminum- or magnesium-containing antacids C. Iron salts D. Calcium

Answer: D

What do you tell patients to do with tablets (antacid)? A. take drugs 30 minutes before meals B. crush it and mix with orange juice C. take 1 hour after meal D. chew thoroughly and take with full glass of water

Answer: C

What does the nurse identify as the principal anti-inflammatory drugs used in the treatment of asthma? A. Theophylline and caffeine B. Ipratropium and tiotropium C. Glucocorticoids and cromolyn D. Zafirlukast and montelukast

Answer: C Rationale: Sucralfate (Carafate) affects the gastric mucosa. It forms a paste-like substance in the stomach, which adheres to the gastric lining, protecting against adverse effects related to gastric acid. It also stimulates healing of any ulcerated areas of the gastric mucosa.

What information will the nurse include in a teaching plan for the patient who is prescribed sucralfate (Carafate)? A "This medication will neutralize gastric acid." B "This medication will enhance gastric absorption of meals." C "This medication will form a protective barrier over the gastric mucosa." D "This medication will inhibit gastric acid."

Answer: C Rationale: Anticholinergic agents block the parasympathetic nervous system, which causes the body to "rest and digest." Blocking of these effects leads to constipation, urinary retention, and decreased secretions (dry mouth).

What instruction is most important for the nurse to teach a patient who is taking an anticholinergic agent to treat nausea and vomiting? A "Assess your stools for dark streaks." B "Do not take more than two doses of this medication." C "Brush your teeth and gargle to help with dryness in your mouth." D "Check your heart rate and call the health care provider if it gets below 50 beats/min."

Answer: A Rationale: Nitrofurantoin carries a significant drug alert. The patient should be taught to avoid sunlight, use sunscreen, and wear protective clothing. The patent should notify the health care provider if fever, chills, cough, chest pain, dyspnea, rash, or numbness or tingling of fingers or toes develop.

What instructions would you give to a patient being treated with nitrofurantoin (Furadantin, Macrodantin)? A. Avoid sunlight and wear sunscreen or use protective clothing. B. Avoid concurrent consumption of calcium-containing products. C. Take the medication concurrently with vitamin D. Do not be alarmed if urine is red.

Answer: A Rationale: Ranitidine (Zantac) should be given just before meals to decrease food-induced acid secretion or at bedtime.

What is a priority nursing intervention when administering ranitidine (Zantac)? A Administer just before meals. B Administer right after eating. C Administer 1 to 2 hours after meals. D Administer during meals.

Answer: C Rationale: Lactulose reduces blood ammonia levels by forcing ammonia from the blood into the colon. It is useful in treating patients with hepatic encephalopathy.

What is an additional function of lactulose, a hyperosmotic laxative? A. Decreasing peripheral edema B. Correcting sodium imbalances C. Reducing ammonia levels D. Alleviating galactose intolerance

Answer: B

What is the major difference between the different classes of cephalosporins? A. pharmacodynamics B. spectrum of activity C. emergence of drug resistance D. tendency to induce allergic reactions

Answer: B Rationale: Loratadine (Claritin) does not affect the central nervous system and therefore is nonsedating. There is insufficient evidence to indicate that loratadine (Claritin) can cause dysrhythmias, can act as a bronchodilator, or cause gastrointestinal upset than other comparable medications.

What is the most important thing for the nurse to teach a patient who is switching allergy medications from diphenhydramine (Benadryl) to loratadine (Claritin)? A Loratadine can potentially cause dysrhythmias. B Loratadine has fewer sedative effects. C Loratadine has increased bronchodilating effects. D Loratadine causes less gastrointestinal upset.

Answer: B Rationale: Loratadine (Claritin) does not affect the central nervous system and therefore is nonsedating. There is insufficient evidence to indicate that loratadine (Claritin) can cause dysrhythmias, can act as a bronchodilator, or cause gastrointestinal upset than other comparable medications.

What is the most important thing for the nurse to teach a patient who is switching allergy medications from diphenhydramine (Benadryl) to loratadine (Claritin)? A Loratadine can potentially cause dysrhythmias. B Loratadine has fewer sedative effects. C Loratadine has increased bronchodilating effects. D Loratadine causes less gastrointestinal upset.

Monitor blood glucose levels every 4 hours when taking albuterol. Rationale:Beta2 agonists may increase blood glucose levels. Clients with diabetes should monitor serum glucose levels frequently while taking this medication.

What is the most important thing for the nurse to teach the client with a history of diabetes and asthma who has started on albuterol PRN?

Answer: D

What is the role of corticosteroids in the treatment of acute respiratory disorders? A. They stimulate the immune system. B. They directly dilate the bronchi. C. They increase gas exchange in the alveoli. D. They decrease inflammation.

Answer: D Rationale: St. John's wort has been shown to decrease serum montelukast (Singulair) levels. The other substances do not interact with montelukast.

What over-the-counter product will the nurse instruct the patient to avoid when taking montelukast (Singulair)? A Acetaminophen (Tylenol) B Echinacea C Diphenhydramine (Benadryl) D St. John's wort

Answer: D Rationale: St. John's wort has been shown to decrease serum montelukast (Singulair) levels. The other substances do not interact with montelukast.

What over-the-counter product will the nurse instruct the patient to avoid when taking montelukast (Singulair)? A Acetaminophen (Tylenol) B Echinacea C Diphenhydramine (Benadryl) D St. John's wort

Answer: A, B Rationale: When a bronchodilator and a glucocorticoid inhaler are given together, the bronchodilator is given first. The nurse should wait for 5 minutes before administering the glucocorticoid. This drug combination is more effective than when given alone.

What should the nurse be aware of when ipratropium, a bronchodilator, and a glucocorticoid inhaler are ordered together? (Select all that apply.) A. The bronchodilator is given 5 minutes before a glucocorticoid. B. The drugs are more effective in combination than when given alone. C. The glucocorticoid is administered 10 minutes before the bronchodilator. D. The glucocorticoid should be administered immediately after the bronchodilator.

Answer: A,B,D,F

What should the nurse include when teaching a client about gentamicin (Garamycin)? (Select all that apply.) a.Client should report any hearing loss. b.Client must use sunscreen. c.IV gentamicin will be given over 20 minutes. d.Client will be monitored for mouth ulcers and vaginitis. e.Peak levels will be drawn 30 minutes prior to IV dose. f.Client should increase fluid intake.

d. Take each dose with a full glass of water or other liquid. Docusate lowers the surface tension of stool, permitting water and fats to penetrate and soften the stool for easier passage. The patient should take the dose with a full glass of water and should increase overall fluid intake, if able, to enhance effectiveness of the medication. Dietary fiber intake should be a minimum of 20 g daily to prevent constipation. Mineral oil and extra salt are not recommended.

What should the nurse instruct the patient to do to best enhance the effectiveness of a daily dose of docusate sodium (Colace)? a. Take a dose of mineral oil at the same time. b. Add extra salt to food on at least one meal tray. c. Ensure dietary intake of 10 g of fiber each day. d. Take each dose with a full glass of water or other liquid.

Answer: D

What statement by the patient, who is using the anticholinergic inhaler ipratropium bromide (Atrovent), indicates to the nurse that teaching has been successful? A. "I will increase my intake of caffeine." B. "I may gain weight as a result of taking this medication." C. "Nausea and vomiting are common adverse effects of this medication, so I will always take it with meals." D. "This inhaler is not to be used alone to treat an acute asthma attack."

Answer: B

What system assessment will the nurse monitor in a patient due to a known common side effect of traditional antihistamines? A.Cardiac status B. Neurologic status C. Respiratory status D. Gastrointestinal status

Answer: C Rationale: Acetylcysteine is a mucolytic drug used to liquefy and loosen bronchial secretions in order to enhance their expectoration.

What will the nurse expect to find that would indicate a therapeutic effect of acetylcysteine (Mucomyst)? A Decreased cough reflex B Decreased nasal secretions C Liquefying and loosening of bronchial secretions D Relief of bronchospasms

Answer: C Rationale: Acetylcysteine is a mucolytic drug used to liquefy and loosen bronchial secretions in order to enhance their expectoration.

What will the nurse expect to find that would indicate a therapeutic effect of acetylcysteine (Mucomyst)? A Decreased cough reflex B Decreased nasal secretions C Liquefying and loosening of bronchial secretions Correct D Relief of bronchospasms

Answer: C Rationale: Isoniazid (INH) can cause neurotoxicity. Pyridoxine (vitamin B6) is the drug of choice to prevent this adverse reaction. It is not an antiinfective agent and thus will work to destroy the mycobacterium or prevent drug resistance. Vitamin C is not taken with this drug; the drug is appropriate for most patients, and INH with pyridoxine is not multidrug therapy.

What will the nurse teach a patient who is taking isoniazid (INH)? A "You will need to take vitamin C to potentiate the action of INH." B "You should not be on that drug. I will check with the health care provider." C "Pyridoxine (vitamin B6) will prevent numbness and tingling that can occur when taking isoniazid." D "Multidrug therapy is necessary to prevent the occurrence of resistant bacteria."

Answer: C Rationale: Combining antiemetic agents from various categories allows the blocking of the vomiting center and chemoreceptor trigger zone through different pathways, thus enhancing the antiemetic effect.

What will the nurse teach the patient about the reason for administering multiple medications for relief of nausea and vomiting? A Combination therapy decreases the risk of constipation. B Combination therapy is more cost-effective. C Combination therapy blocks different vomiting pathways. D Combination therapy decreases side effects due to lower doses of each drug.

A) Tremors (Extrapyramidal side effects, including tremors and dyskinesias, may occur as a result of metoclopramide (Reglan) administration.)

When administered a dose of metoclopramide (Reglan), a patient complains of nausea. The nurse would teach the patient to report which of the following potential adverse effects? A) Tremors B) Constipation C) Double vision D) Numbness in the fingers and dose

Answer: A

When assessing a patient who is receiving a decongestant, the nurse will recognize that a potential contraindication to this drug would be which condition? a. glaucoma b. fever c. peptic ulcer disease d. allergic rhinitis

Answer: C

When giving dextromethorphan, the nurse understands that this drug suppresses the cough reflex by which mechanism of action? A. Causing depression of the central nervous system B. Anesthetizing the stretch receptors C. Having direct action on the cough center D. Decreasing the viscosity of the bronchial secretions

Answer: D Rationale: Antibiotic therapy can destroy the normal flora of the body, which typically inhibit the overgrowth of fungi and yeast. When the normal flora are decreased, these organisms can overgrow and cause infections.

When instructing a patient about antibiotic therapy, the nurse explains that which condition occurs when the normal flora are disturbed during antibiotic therapy? A Hypersensitivity B Rebound toxicity C Organ toxicity D Superinfection

Answer: C Combining antiemetic drugs from various categories allows the blocking of the vomiting center and chemoreceptor trigger zone through different pathways, thus enhancing the antiemetic effect.

When planning administration of antiemetic medications to a patient, the nurse is aware that combination therapy is preferred because A) the risk of constipation is decreased. B) lower doses of medication are cost-effective. C) different vomiting pathways are blocked. D) increased sedation is achieved by higher doses of medication.

Answer: C Rationale: Forcing fluids will help prevent crystallization in the urine and kidney stone formation associated with sulfonamide antibiotics, regardless of the type of fluid consumed. It is outside the nurse's scope of practice to decide to insert a Foley catheter. Consuming a specific type of diet will not decrease the risk of crystallization.

When planning care for a patient receiving a sulfonamide antibiotic, which is a primary intervention? A Encourage liquids that produce acidic urine. B Encourage a diet that causes an alkaline ash. C Force fluids to at least 2000 mL/day. D Insert a Foley catheter for accurate input and output measurement.

Answer: C Rationale: Trough serum drug levels should be drawn at least 8 to 12 hours after the medication is infused

When planning care for a patient receiving once-daily intravenous gentamicin therapy, the nurse schedules a trough drug level to be drawn A. 30 minutes after beginning the antibiotic infusion. B. 60 minutes after beginning the antibiotic infusion. C. 12 hours after completing the antibiotic infusion. D. 18 hours after completing the antibiotic infusion.

Answer: C Rationale: Metoclopramide should be administered 30 minutes before meals and at bedtime. Administering the medication before meals allows time for onset to increase gastrointestinal motility before food ingestion, thus decreasing stomach distention and resulting nausea and vomiting.

When planning to administer metoclopramide (Reglan), the nurse is aware that this drug must be given: A) with food to decrease gastrointestinal upset. B) every 6 hours around the clock. C) 30 minutes before meals and at bedtime. D) with a full glass of water first thing in the morning.

A. "Avoid taking your loratadine (Claritin) and triamcinolone (Azmacort) inhaler for 5 days before testing."

When preparing the client for allergy testing, the nurse provides the client with which instruction? A. "Avoid taking your loratadine (Claritin) and triamcinolone (Azmacort) inhaler for 5 days before testing." B. "It is okay to use your fluticasone propionate (Flonase) nasal spray before testing." C. "Aspirin in a low dose is allowed to be taken before testing." D. "You can take antihistamine nasal sprays before testing."

Answer: C

When reviewing the various types of antinausea medications, the nurse recognizes that prokinetic drugs are also used for... A. Motion sickness B. Vertigo C. Delayed gastric emptying D. GI obstruction

ANSWER: A Rationale: A major disadvantage of cimetidine is that it inhibits the cytochrome P450 drug-metabolizing system in the liver, decreasing hepatic metabolism of numerous other drugs, including warfarin, thereby increasing blood levels and risks of toxicity. Other histamine2 receptor antagonists, such as ranitidine, do not affect the cytochrome P450 drug-metabolizing system; therefore, they do not interfere with the metabolism of other drugs. They are safer to use in patients who are on multiple medications, especially those that are known to be metabolized by the same cytochrome P450 system and those with narrow therapeutic windows.

When taking a patient's history, the nurse notes that the patient is taking warfarin and cimetidine—a histamine2 receptor antagonist—concurrently. The nurse should anticipate that the A. Warfarin effects would be increased B. Cimetidine effects would be increased C. Warfarin effects would be decreased D. Cimetidine effects would be decreased

Answer: CDF

When teaching a patient who will be receiving antihistamines, the nurse will include which instructions? (Select all that apply.) A. "Antihistamines are generally safe to take with over-the-counter medications." B. "Take the medication on an empty stomach to maximize absorption of the drug." C. "Take the medication with food to minimize gastrointestinal distress." D. "Drink extra fluids if possible." E. "Antihistamines may cause restlessness and disturbed sleep." F. "Avoid activities that require alertness until you know how adverse effects are tolerated."

Answer: A

When the nurse teaches a patient about taking bisacodyl tablets, which instruction is correct? A. "Take this medication on an empty stomach." B. "Chew the tablet for quicker onset." C. "Take this medication with juice or milk." D. "Take this medication with an antacid if it upsets your stomach."

Answer: C Rationale: Fluid and electrolyte imbalance is a serious complication of the therapy. The patient needs to be monitored for potential problems.

Which assessment is most important for the patient who is taking stimulant laxatives? A Monitor bowel elimination daily. B Monitor intake and output. C Monitor signs and symptoms of fluid and electrolyte imbalance. D Monitor heart rate and blood pressure every 4 hours.

ANS: B Rationale: Most decongestants work by increasing blood vessel constriction. This action increases peripheral vascular resistance and blood pressure. The client who already has hypertension may develop dangerously high blood pressure when taking a decongestant. The client who has a latex allergy, taking oral contraceptives, or has type 1 diabetes would not be likely to be as affected by the decongestant in such a life-threatening manner as the client who is hypertensive.

Which client will the nurse caution to avoid taking over-the-counter decongestants for manifestations of a cold or flu? a. A young man with a latex allergy b. A middle-aged woman with hypertension c. A teenage woman who is taking oral contraceptives d. An older man who has had type 1 diabetes mellitus for 20 years

D -Ondansetron = serotonin blocker -Metoclopramide = prokinetic drug -meclizine = antihistamine -droperidol = antidopaminergic drug

Which drug works by blocking serotonin in the gastrointestinal tract, vomiting center, and chemoreceptor trigger zone (CTZ)? A) metoclopramide (Reglan) B) meclizine (Antivert) C) droperidol (Inapsine) D) ondansetron (Zofran)

C. Cromolyn is best suited for prophylaxis and should be given before symptoms start, because responses may take a week or 2 to develop. It is important for the patient to know about cromolyn's moderate effectiveness, few adverse reactions, and suppression of histamine release, but it is more important that the patient be informed of the delay in response.

Which instruction by the nurse should be the priority for a patient scheduled to start intranasal cromolyn (NasalCrom)? A. "It is only moderately effective." B. "There are few adverse reactions." C. "Relief may take a week or two." D. "It suppresses histamine release."

Answer: Use sunscreen and protective clothing when outdoors Rationale: Photosensitivity is a common side effect of tetracycline. Exposure to the sun can cause severe burns. The medication should not be taken with milk and should be completely finished

Which instruction will the nurse include in the discharge teaching for a patient receiving tetracycline?

Answer: B Rationale: Photosensitivity is a common side effect of tetracycline. Exposure to the sun can cause severe burns. The medication should not be taken with milk and should be completely finished.

Which instruction will the nurse include in the discharge teaching for a patient receiving tetracycline? A "Take the medication until you feel better." B "Use sunscreen and protective clothing when outdoors." C "Keep the remainder of the medication in case of recurrence." D "Take the medication with food or milk to minimize gastrointestinal upset."

Answer: C Rationale: Holding the breath for 10 seconds allows the medication to be absorbed in the bronchial tree rather than be immediately exhaled.

Which instruction will the nurse include when teaching a patient about the proper use of metered-dose inhalers? A "After you inhale the medication once, repeat until you obtain relief." B "Make sure that you puff out air repeatedly after you inhale the medication." C "Hold your breath for 10 seconds if you can after you inhale the medication." D "Hold the inhaler in your mouth, take a deep breath, and then compress the inhaler."

Answer: A, D, E, F RATIONALE: Increased fluid intake and cranberry juice (A) are recommended for prevention and treatment of urinary tract infections, which frequently accompany vaginal infections. It is not necessary to taper use of this drug (B) or to check the blood pressure daily (C), as this condition is not related to hypertension. Flagyl can cause a disulfiram-like reaction if taken in conjunction with ingestion of alcohol, so the client should be instructed to avoid alcohol (D). All sexual partners should be treated at the same time (E) and condoms should be used until after treatment is completed to avoid reinfection (F).

Which instruction(s) should the nurse give to a female client who just received a prescription for oral metronidazole (Flagyl) for treatment of trichomonas vaginalis? (Select all that apply.) A) Increase fluid intake, especially cranberry juice. B) Do not abruptly discontinue the medication; taper use. C) Check blood pressure daily to detect hypertension. D) Avoid drinking alcohol while taking this medication. E) Use condoms until treatment is completed. F) Ensure that all sexual partners are treated at the same time. A) Increase fluid intake, especially cranberry juice. D) Avoid drinking alcohol while taking this medication. E) Use condoms until treatment is completed. F) Ensure that all sexual partners are treated at the same time.

ANS: B Teach the client to increase fluid intake to more than 10 glasses of water or juice daily unless another medical problem requires fluid restriction. Treatment of sinusitis includes the use of broad-spectrum antibiotics (e.g., amoxicillin), analgesics for pain and fever, decongestants, steam humidification, hot and wet packs over the sinus area, and nasal saline irrigations. If this treatment plan is not successful, the client may need to be evaluated with sinus films and computed tomography (CT). Surgical intervention may be needed.

Which is the highest priority teaching need for a client with sinusitis? a. Use cold packs over the sinus area. b. Increase his fluid intake to more than 10 glasses of fluid daily. c. Avoid using nasal saline irrigations because of the risk of spreading the infection. d. Keep the room air dehumidified to dry out the nasal sinuses.

Answer: A,B,C

Which medications may be administered by the nurse for infections in a patient with a severe penicillin allergy? (Select all that apply.) A. Vancomycin B. Erythromycin C. Clindamycin D. Amoxicillin

Answer: C Rationale: Adequate renal function is needed to excrete excess magnesium. This intervention is essential to predict how the patient will handle the therapy. The other interventions are important, but assessing renal function is the priority.

Which nursing intervention is a priority before administering magnesium hydroxide to a patient? A Obtain a history of constipation and causes. B Record baseline vital signs. C Assess renal function. D Advise the patient to take the medication with a glass of water.

Answer: C Rationale: Beta-adrenergic drugs such as albuterol are most often used for rapid bronchodilation

Which of the following drugs is most immediately helpful in treating a severe acute asthma attack? a. Beclometasone (Qvar) b. Zileuton (Zyflo ZR) c. Albuterol (Proventil, Ventolin) d. Salmeterol (Serevent)

ANSWER: B → Magnesium hydroxide increases osmotic pressure in the intestinal lumen and causes water to be retained.

Which of the following mechanism of action is shown by over-the-counter drug magnesium hydroxide (milk of magnesia)? A. Increased gastric secretions B. Increased osmotic pressure in the intestinal lumen C. Binding to diarrhea-causing bacteria for excretion D. Decreased gastrointestinal motility

ANSWER: D → Bulk-forming laxatives such as methylcellulose and psyllium are contraindicated for patients with dysphagia because they may cause obstruction. Patients with dysphagia are also not good candidates for bulk-forming laxatives, because they may have decreased fluid intake secondary to their dysphagia. Mineral oil in oral forma- tion is contraindicated for these patients because it may be aspirated leading to a lipid pneumonia. Docusate sodium is the safest choice for this patient.

Which of the following oral medications is safe to use in a 60-year-old constipated patient with dysphagia? A. methylcellulose (Citrucel) B. psyllium (Metamucil) C. mineral oil (Agoral Plain) D. docusate sodium (Colace)

Answer: 3 *Don't take tetracyclines with dairy products, antacids, or laxatives!

Which of the following statements by your patient reflects an understanding about how to take their doxycycline? 1. "I will take it with milk." 2. "I will take it with milk of magnesia." 3. "I will take it with a glass of iced tea." 4. "I will take it with a bourbon on the rocks."

Answer: C

Which over-the-counter/herbal product, when taken with theophylline, can decrease theophylline's serum drug levels? A. Caffeine B. Diphenhydramine C. St. John's wort D. Echinacea

d. A 32-year-old patient who is receiving ciprofloxacin (Cipro) for 3 weeks Oral candidiasis is caused by prolonged antibiotic treatment (e.g., ciprofloxacin) or high doses of corticosteroids. Amphotericin B is used to treat candidiasis. Vitamin B and C deficiencies are rare but may lead to Vincent's infection. Use of tobacco products leads to stomatitis.

Which patients would be at highest risk for developing oral candidiasis? a. A 74-year-old patient who has vitamin B and C deficiencies b. A 22-year-old patient who smokes 2 packs of cigarettes per day c. A 58-year-old patient who is receiving amphotericin B for 2 days d. A 32-year-old patient who is receiving ciprofloxacin (Cipro) for 3 weeks

Answer: B

Which statement by a patient best demonstrates an understanding of the teaching on flunisolide (AeroBid)? A. "I will take two puffs to treat an acute asthma attack." B. "I will rinse my mouth with water after each use." C. "I will immediately stop taking my oral prednisone as soon as I start using the AeroBid." D. "I will not use my albuterol inhaler while I am taking AeroBid."

Answer: C Rationale: Transdermal scopolamine patches should be applied to non-irritated areas behind the ear, not on the arms. It is appropriate for the patient to refrain from driving while utilizing antiemetics, to use Tylenol to treat headaches caused by ondansetron (Zofran), and to take antiemetics preventatively prior to and after chemotherapy.

Which statement by the patient indicates that further teaching is needed about antiemetic medication? A "I will not drive while I am taking these medications because they may cause drowsiness." B "I may take Tylenol to treat the headache caused by ondansetron (Zofran)." C "I will apply the scopolamine patches to rotating sites on my upper arms." D "I should take my prescribed antiemetic before receiving my chemotherapy dose and continue afterwards."

Answer: B Rationale: Patients taking histamine2-receptor blocking agents should avoid spicy foods, extremes in temperatures, alcohol, and smoking. They should also increase bulk and fluids in their diets to prevent constipation. The medications should be taken with meals, not after meals.

Which statement demonstrates to the nurse that the patient understands instructions regarding the use of histamine2-receptor antagonists? A "Since I am taking this medication, it is all right for me to eat spicy foods." B "Smoking decreases the effects of the medication, so I should try a cessation program." C "I should take this medication 1 hour after each meal to decrease gastric acidity." D "I should decrease bulk and fluids in my diet to prevent diarrhea."

ANS: C Washing the dry powder inhaler (DPI) may cause the medication in the inhaler to clump together. This action reduces the precision of the delivery of the drug to the client. The other statements are all correct—the client should not exhale into the inhaler, can store the inhaler in his or her bedroom, and will need to inhale more forcefully than with an aerosol inhaler

Which statement indicates that the client needs additional teaching about a dry powder inhaler? a. "I will not exhale into the inhaler." b. "I will store the inhaler in the drawer of my bedroom dresser." c. "I will wash the inhaler mouthpiece daily with soap and water." d. "I will inhale twice as hard through this inhaler as I do with my aerosol inhaler."

ANS: B Corticosteroids decrease inflammatory and immune responses in many ways, including preventing the synthesis of mediators. Both inhaled corticosteroids and those taken orally are preventive; they are not effective in reversing symptoms during an asthma attack and should not be used as rescue drugs. Systemic corticosteroids, because of severe side effects, are avoided for mild to moderate intermittent asthma and are used on a short-term basis for moderate asthma.

Which statement indicates that the client understands teaching about the correct use of a corticosteroid medication? a. "This drug can reverse my symptoms during an asthma attack." b. "This drug is effective in decreasing the frequency of my asthma attacks." c. "This drug can be used most effectively as a rescue agent." d. "This drug can safely be used on a long-term basis for multiple applications daily."

ANS: C This medication will help prevent an acute asthma attack because it is long acting. The client will take this medication every day for best effect. This is not the medication the client will use during an acute asthma attack because it does not have an immediate onset of action. The client will not be weaned off this medication because this is likely to be one of his daily medications.

Which statement indicates that the client understands teaching about the use of his long-acting beta2 agonist medication? a. "I will not have to take this medication every day." b. "I will take an extra dose of this medication when I have an asthma attack." c. "I will take this medication daily to prevent an acute attack." d. "I will eventually be able to stop using this medication."

Answer: B Rationale: Taking this medication with food will help minimize gastrointestinal upset. Ketoconazole (Nizoral) should not be taken with coffee, tea, or acidic fruit juices. Additionally, it needs to be taken at least 2 hours before or after the ingestion of alkaline products or antacids. Awarded 1.0 points out of 1.0 possible points.

Which statement indicates to the nurse that the patient understands the medication instructions regarding ketoconazole (Nizoral) for treatment of candidiasis? A "I will take this medication with orange juice for better absorption." B "I need to take this drug with food to minimize gastrointestinal distress." C "I can take this medication with antacids if it causes gastrointestinal discomfort." D "I can expect my skin to turn yellow from taking this drug."

Answer: D Rationale: The patient should not just keep taking extra puffs of the inhaler to make breathing easier. Excessive treatment could trigger paradoxical bronchospasm, which would worsen the patient's respiratory status. Rinsing the mouth after the puffs will eliminate a bad taste. Waiting 1 to 2 minutes between each puff will facilitate the effectiveness of the administration. Ipratropium is not used in an emergency for COPD.

Which statement made by the patient with chronic obstructive pulmonary disease (COPD) indicates a need for further teaching regarding the use of an ipratropium inhaler? A. "I can rinse my mouth following the two puffs to get rid of the bad taste." B. "I should wait at least 1 to 2 minutes between each puff of the inhaler." C. "Because this medication is not fast-acting, I cannot use it in an emergency if my breathing gets worse." D. "If my breathing gets worse, I should keep taking extra puffs of the inhaler until I can breathe more easily."

Answer: C Rationale: Transdermal scopolamine patches should be applied to nonirritated areas behind the ear, not on the arms.

Which statement regarding antiemetic medications, if made by the patient, indicates the need for further patient teaching? A) "I will not drive while I am taking these medications because they may cause drowsiness." B) "I may take Tylenol to treat the headache caused by ondansetron." C) "I will apply the scopolamine patches to rotating sites on my arms." D) "I should take my prescribed antiemetic before receiving my chemotherapy dose."

Answer: C

Which statement would be included when teaching a patient about the proper use of metered-dose inhalers? A. "After you inhale the medication once, repeat until you obtain relief." B. "Make sure that you puff out air repeatedly after you inhale the medication." C. "Wait 1 to 2 minutes before you take a second puff of the same drug." D. "Hold the inhaler in your mouth, take a deep breath, and then compress the inhaler."

Answer: D Rationale: The client should be instructed to report loose stools or diarrhea because of possible pseudomembranous colitis. Photosensitivity during sun exposure is common with tetracyclines, which should not be given to children less than 8 years of age; the drugs are affected by light and extreme heat.

Which teaching has highest priority for the client taking azithromycin (Zithromax)? A. Instruct the client to use sunblock and protective clothing during sun exposure. B. Instruct the client to store the drug out of light and extreme heat. C. Inform parents that children younger than 8 years should not take the drug, to avoid teeth discoloration. D. Instruct the client to report any loose stools or diarrhea.

Answer: A,B,D Rationale: Not completing a full course of antibiotic therapy can allow bacteria that are not killed but have been exposed to the antibiotic to adapt their physiology to become resistant to that antibiotic. The same thing can occur when bacteria are exposed to antibiotics in the environment or when antibiotics are erroneously used to treat a viral infection.

While instructing a patient about antibiotic therapy, the nurse explains to the patient that bacterial resistance to antibiotics can occur when what happens? (Select all that apply.) A. Patients stop taking an antibiotic after they feel better. B. Environmental dispersion of antibiotic liquid occurs. C. Antibiotics are prescribed according to culture and sensitivity reports. D. Antibiotics are prescribed to treat a viral infection. E. Antibiotics are taken with water or juice. F. Antibiotics are taken with ascorbic acid (vitamin C).

c. Antibiotic(s), proton pump inhibitor, and bismuth To eradicate H. pylori, a combination of antibiotics, a proton pump inhibitor, and possibly bismuth (for quadruple therapy) will be used. Corticosteroids, aspirin, and NSAIDs are drugs that can cause gastritis and do not affect H. pylori.

he patient with chronic gastritis is being put on a combination of medications to eradicate H. pylori. Which drugs does the nurse know will probably be used for this patient? a. Antibiotic(s), antacid, and corticosteroid b. Antibiotic(s), aspirin, and antiulcer/protectant c. Antibiotic(s), proton pump inhibitor, and bismuth d.Antibiotic(s) and nonsteroidal antiinflammatory drugs (NSAIDs)


Kaugnay na mga set ng pag-aaral

AP Chemistry~First Semester Final Examination & Entailment (Nolan)~(2020-2021)

View Set

Prep U 47 Management of Patients With Gastric and Duodenal Disorders

View Set

8. Third month to Birth: The fetus and placenta

View Set

Chapter 45 - Hormones and the Endocrine System

View Set

Chapter 12: Oncologic Management

View Set